Chuyên đề hình học phẳng bồi dưỡng học sinh giỏi toán

Chuyên đề hình học phẳng bồi dưỡng học sinh giỏi toán

4.8/5

Tác giả: Thầy Tùng

Đăng ngày: 22 Aug 2022

Lưu về Facebook:
Hình minh họa Chuyên đề hình học phẳng bồi dưỡng học sinh giỏi toán

Công thức toán học không thể tải, để xem trọn bộ tài liệu hoặc in ra làm bài tập, hãy tải file word về máy bạn nhé

Loại 1: Chứng minh tính chất: thẳng hàng, đồng quy, song song, vuông góc.

  1. [Trường THPT Chuyên Lê Hồng Phong Nam Định- năm 2015- Tỉnh Nam Định]

Cho hai đường tròn và cắt nhau tại . lần lượt là các đường kính của và . Gọi là trung điểm của ; là điểm thuộc đường phân giác của góc sao cho không vuông góc với và không thuộc hai đường tròn. Đường thẳng đi qua vuông góc với lần lượt cắt các đường tròn , tại các điểm khác . cắt đường tròn tại điểm thứ hai , cắt đường tròn tại điểm thứ hai .

1. Gọi là giao điểm của với . Chứng minh rằng là tiếp tuyến của đường tròn .

2. Chứng minh rằng 3 đường thẳng đồng quy.

Lời giải

1. Không mất tính tổng quát giả sử là điểm thuộc đường phân giác trong của góc .

Ta có tứ giác là hình bình hành nên suy ra

Lại có

Do đó thẳng hàng. Chứng minh tương tự ta có thẳng hàng Mặt khác

Do đó là tiếp tuyến của đường tròn

2. Ta có nên 4 điểm cùng thuộc đường tròn đường kính .

Mà nên suy ra là tiếp tuyến của đường tròn đường kính .

Do đó (1)

Mặt khác

(2)

Từ (1) và (2) suy ra

Vậy 4 điểm cùng thuộc một đường tròn.

Gọi là giao điểm của và

Vì 4 điểm cùng thuộc một đường tròn nên ta có

(3)

Ta có (4)

Gọi là giao điểm của với

Chứng minh tương tự câu 1) ta có là tiếp tuyến của đường tròn

Mặt khác tứ giác là hình thang vuông tại và là trung điểm của nên suy ra . Do đó (5)

Từ (3), (4), (5) suy ra cùng thuộc trục đẳng phương của hai đường tròn nên thẳng hàng. Vậy 3 đường thẳng đồng quy tại

*) Chú ý: Nếu HS không sử dụng góc định hướng thì phải xét các trường hợp vị trí của điểm ( nằm ngoài các đoạn và nằm trong các đoạn )

  1. [Trường THPT Lương Văn Tụy- Ninh Bình- Vòng 2]

Cho tam giác nhọn có trực tâm ., là trung điểm của . Các đường phân giác của góc , cắt nhau tại . Chứng minh:

a, Góc là góc vuông

b, thẳng hàng.

  1. [SỞ Bình Định- năm học 2012-2013]

Trong tam giác , là chân đường vuông góc hạ từ xuống đường phân giác trong của góc lần lượt là chân đường vuông góc hạ từ các đỉnh xuống đường phân giác trong của góc . Gọi là giao điểm của các đường thẳng và , là giao điểm của các đường thẳng và , là giao điểm của các đường thẳng và . Chứng minh rằng .

  1. [Trường THPT Chuyên Hoàng Văn Thụ - năm 2015- Tỉnh Hòa Bình]

Cho và hai đường tròn tiếp xúc ngoài với nhau và tiếp xúc trong với . Gọi là tiếp điểm của và ; là tiếp điểm của với . Tiếp tuyến chung tại của cắt tại A. cắt tại ; cắt tại .

Chứng minh rằng .

cắt ở ; cắt tại . Chứng minh rằng là tâm đường tròn nội tiếp của tam giác .

Chứng minh rằng đồng quy.

Lời giải

a) thuộc trục đẳng phương của và nên suy ra là tứ giác nội tiếp dẫn đến

b) Gọi là giao điểm của với .

Tam giác vuông tại có là đường cao

là tứ giác nội tiếp

Suy ra A là tâm đường tròn ngoại tiếp tam giác

Dẫn đến

Suy ra là phân giác của

Rõ ràng là phân giác của (do )

Vì thế là tâm đường tròn nội tiếp tam giác

c)Giả sử cắt tại , gọi là bán kính của , .

Rõ ràng là tâm vị tự ngoài của và ,lại có

Suy ra

Dẫn đến thẳng hàng (Menelauyt đảo)

Vậy đồng quy.

  1. [ ĐỀ THI CHỌN HỌC SINH GIỎI TRẠI HÈ HÙNG VƯƠNG LẦN THỨ XI- CHUYÊN HẠ LONG]

Cho hai đường tròn và với cắt nhau tại hai điểm phân biệt . Một đường thẳng d tiếp xúc với đường tròn và lần lượt tại và . Gọi và lần lượt là chân đường vuông góc hạ từ và xuống .Các đường thẳngvà cắt các đường tròn tại và.Chứng minh rằng thẳng hàng.

Hướng dẫn giải

Gọi là giao điểm của và , khi đó là tâm vị tự ngoài của hai đường tròn và . Đặt , khi đó ta có:

Gọi là giao điểm của với và . Khi đó ta có:

Mà// // nên

Suy ra là trung trực của .

Mà là trung trực của . Vậy tứ giác là hình thoi

Do đó// hay// .

Giả sử biến thành khi đó//

Mà thuộc suy ra thuộc do đó .

Vậy biến thành .

Tương tự ta có biến thành . Suy ra thẳng hàng.

  1. Cho đường tròn nội tiếp tiếp xúc với các cạnh tương ứng tại . Đường thẳng cắt tại . Đường tròn đường kính cắt tại (). Gọi () tương ứng là giao của với . Hai đường thẳng và cắt nhau tại . Đường tròn cắt tại và đường tròn cắt tại . Chứng minh rằng đồng quy.

Hướng dẫn giải

Gọi là trung điểm đoạn . Ta có , do đó điều này suy ra là tiếp tuyến. Do đó .

Mặt khác là tiếp tuyến của , do đó cũng là tiếp tuyến của .

Vì vậy

Suy ra đi qua trung điểm của đoạn ( bổ đề quen thuộc trong hình thang ).

Từ đây suy ra là 3 đường trung tuyến của , suy ra ĐPCM.

  1. [KỲ THI CHỌN HỌC SINH GIỎI KHU VỰC DUYÊN HẢI VÀ ĐỒNG BẰNG BẮC BỘ NĂM HỌC 2016 - 2017 ]

Cho tam giác có . Đường tròn nội tiếp tiếp xúc với và tạitương ứng. Gọi là trung điểm của và là điểm đối xứng với qua . Đường thẳng vuông góc với tại cắt tại là giao điểm thứ hai của với . Chứng minh rằng .

  1. [ĐỀ XUẤT ĐỀ THI DUYÊN HẢI BẮC BỘ.Trường THPT Chuyên Hoàng Văn Thụ - Tỉnh Hòa Bình. Năm học 2012-2013]

Cho tam giác . Các phân giác ngoài của các góc lần lượt cắt cạnh đối diện tại của tam giác tại . CMR thẳng hàng và thuộc đường thẳng vuông góc với ở đây lần lượt là tâm đường tròn ngoại tiếp và nội tiếp của tam giác

Hướng dẫn giải

Qua kẻ các đường thẳng vuông góc lần lượt cắt .

chung

trục đt của và

là tâm đường tròn bàng tiếp của thẳng hàng.

C

thẳng hàng và

  1. [Đề xuất lớp 11 Sở GD- ĐT Quảng Ninh- Trường THPT Chuyên Hạ Long]

Giả sử đường tròn nội tiếp tam giác tiếp xúc với các cạnh theo thứ tự . Đường thẳng qua và song song với cắt tại . Gọi là trung điểm của . Chứng minh rằng: .

Hướng dẫn giải

Gọi là giao điểm của và . QuaN kẻ đường thẳng // cắt , theo thứ tự từ . Vì hai tứ giácvà nội tiếp nên

Mặt khác . Do đó cân tại . Vậy là trung điểm của thẳng hàng.

Lại có là trực tâm

Gọi là giao điểm của và

là giao điểm của và

Mà nội tiếp nên(Đpcm).

  1. [ĐỀ NGHỊ THI CHỌN HSG VÙNG DUYÊN HẢI BẮC BỘ LỚP 11 - Tr­êng T.H.P.T Chuyªn Th¸i B×nh.N¨m häc 2013-2014 ]

Cho tam giác vuông tại . Hình chữ nhật thay đổi sao cho thuộc , thuộc và thuộc .Chứng minh rằng.

1)

2) luôn đi qua một điểm cố định.

Hướng dẫn giải

1) Lấy theo thứ tự thuộc sao cho (h.1).

(h.2.1)

Ta có:

Do đó các tam giác đồng dạng.

Vậy

2) Đặt (h.2.2).

Theo định lí Pappus: thẳng hàng .

Gọi là hình chiếu của trên ; theo thứ tự là trung điểm của , .

Dễ thấy thẳng hàng; thẳng hàng; .

Vậy

Do đó thẳng hàng.

Từ và suy ra đi qua (đpcm).

(h.2.2)

  1. [ ĐỀ THI ĐỀ XUẤT TRẠI HÈ HÙNG VƯƠNG LẦN THỨ XI-TRƯỜNG THPT CHUYÊN TUYÊN QUANG ]

Cho tam giác với . Các đường trung tuyến và phân giác trong góc cắt tại và tương ứng. Đường thẳng qua vuông góc với cắt lần lượt tại và ; đường thẳng qua vuông góc với cắt đường thẳng tại . Chứng minh vuông góc với .

Hướng dẫn giải

Gọi là giao điểm thứ hai của với đường tròn ngoại tiếp tam giác , dễ thấy suy ra vuông góc với .Đặt và xét phép vị tự . Khi đó thuộc , thuộc và hai tam giác và có các cạnh tương ứng song song.

Gọi là giao điểm của với , ta có

suy ra tứ giác nội tiếp. Từ đó

Như vậy nên K là trung điểm, hay thuộc , suy ra trùng . Do song song với mà vuông góc với nên vuông góc với .

  1. [TRẠI HÈ HÙNG VƯƠNG-ĐỀ THI CHỌN HỌC SINH GIỎI năm 2015.LẦN THỨ XI ]

Cho tam giác có ba góc nhọn, và nội tiếp đường tròn Các đường cao cắt nhau tại Đường thẳng cắt tại Lấy điểm trên sao cho Đường tròn ngoại tiếp tam giác cắt tại Chứng minh rằng

a) Ba điểm thẳng hàng.

b) Đường thẳng vuông góc với đường thẳng

  1. [ Đề 59- TRƯỜNG THPT CHUYÊN VĨNH PHÚC-ĐỀ THI CHỌN HỌC SINH GIỎI VÙNG DUYÊN HẢI VÀ ĐỒNG BẰNG BẮC BỘ LẦN THỨ VIII- NĂM 2015.MÔN TOÁN - LỚP 11 ]

Cho tam giác nhọn không cân, nội tiếp đường tròn . là điểm nằm trong tam giác sao cho . Đường tròn đường kính cắt các cạnh lần lượt tại và cắt đường tròn tại điểm khác . Chứng minh rằng đồng quy.

Hướng dẫn giải

Gọi là đường kính của , dễ thấy thẳng hàng và . Giả sử cắt tại cắt tại .

Theo định lý Desargues để chứng minh (hay ) đồng quy ta chỉ cần chứng minh thẳng hàng. Áp dụng định lý Menelaus ta được:

Dễ thấy tứ giác EFBC nội tiếp nên

Cũng từ nội tiếp suy ra

Tứ giác là hình bình hành suy ra .

Suy ra .

Ta có

Thay vào ta được . Từ đó áp dụng định lý menelaus cho tam giác ta suy ra thẳng hàng. Bài toán được chứng minh.

  1. [ SỞ GIÁO DỤC VÀ ĐÀO TẠO LÂM ĐỒNG- Trường THPT Chuyên BảoLộc-KỲ THI HSG KHU VỰC DH VÀ ĐBBB LẦN THỨ 9ĐỀ THI ĐỀ NGHỊ MÔN: TOÁN; LỚP: 11]

Cho tứ giác nội tiếp có các cặp cạnh đối không song song. Các đường thẳng và cắt nhau tại điểm và các đường chéo và cắt nhau tại . Đường tròn ngoại tiếp các tam giác cắt nhau tại điểm thứ hai . Chứng minh rằng hai đường thẳng và vuông góc.

Hướng dẫn giải

▪ Gọi là giao điểm của và, là tâm đường tròn ngoại tiếp tứ giác .

Ta dùng kí hiệu tương ứng để chỉ đường tròn ngoại tiếp của tam giác , tứ giác .

Ta có lần lượt là trục đẳng phương của các cặp đường tròn và và và nên đồng quy tại hay thẳng hàng.

▪ Không mất tổng quát ta giả sử F nằm giữa và .

Ta có

(ta có thể dùng góc định hướng cho mọi trường hợp).

Suy ra các điểm cùng thuộc một đường tròn ta gọi là .

▪ Tương tự, các điểm cùng thuộc một đường tròn ta gọi là .

Ta có lần lượt là trục đẳng phương của các cặp đường tròn và , , và nên đồng quy tại hay thẳng hàng.

▪ Xét cực và đối cực đối với đường tròn, ta có là đối cực của nên vuông góc với

Mà thẳng hàng; thẳng hàng nên và vuông góc (điều phải chứng minh).

  1. [KỲ THI HỌC SINH GIỎI CÁC TRƯỜNG THPT CHUYÊN KHU VỰC DUYÊN HẢI VÀ ĐỒNG BẰNG BẮC BỘ LẦN THỨ IX, NĂM HỌC 2015 –2016. ĐỀ THI MÔN TOÁN – KHỐI 11 ]

Cho tam giác nội tiếp đường tròn. Tiếp tuyến của tại cắt nhau tại . Gọi là đường thẳng chứa phân giác trong góc của tam giác . Các đường trung trực của các đoạn thẳng cắt lần lượt tại và . Gọi là giao điểm của và , là tâm đường tròn nội tiếp tam giác , là trực tâm của tam giác .

a. Chứng minh ,đối xứng với nhau qua .

b. Chứng minh thẳng hàng.

Hướng dẫn giải

a) Chứng minh là trung trực của

Không mất tính tổng quát ta giả sử bài toán có vị trí tương đối như hình vẽ.

Gọi là trung điểm của , là giao điểm (khác ) của với , là trung điểm của .

Vì hai tam giác cân nên dễ thấy:

Suy ra, tam giác cân tại và tam giác cân tại . Vậy là trung trực của .

+) Chứng minh ,đối xứng với nhau qua

Ta có:

Vậy hai điểm và đối xứng với nhau qua .

b) Chứng minh đối xứng với nhau qua

Gọi là đường kính của .

Ta có nên mà và vuông góc với nhau suy ra là phân giác góc .

Vậy đối xứng với nhau qua .

+) Dựa vào tính chất của phép đối xứng trục d ta thấy thẳng hàng khi và chỉ khi thẳng hàng. Ta dùng Melenauyt với tam giác để chứng minh điều này.

Ta có điều phải chứng minh.

  1. [HỘI CÁC TRƯỜNG THPT CHUYÊN VÙNG DUYÊN HẢI VÀ ĐỒNG BẰNG BẮC BỘ- TRƯỜNG THPT CHUYÊN BIÊN HÒA-2006]

Cho tam giác với là trực tâm tam giác, là tâm đường tròn ngoại tiếp và là bán kính đường tròn ngoại tiếp. Gọi là điểm đối xứng của qua , là điểm đối xứng của qua , là điểm đối xứng của qua . Chứng minh rằng thẳng hàng khi và chỉ khi .

Hướng dẫn giải

Gọi là trọng tâm tam giác . lần lượt là trung điểm . Gọi là tam giác nhận là trung điểm các cạnh . Do đó là trọng tâm tam giác .

Từ cách dựng suy ra lần lượt là đường trung trực của . Do đó là tâm đường tròn ngoại tiếp tam giác IJK có bán kinh 2R.

Gọi là hình chiếu vuông góc của lên các đường . Do là trọng tâm hai tam giác trên nên:

Xét biến thành

Có là trung điểm nên , . Vậy thẳng hàng. Do đó .

Suy ra và

Vậy thẳng hàng và biến thành

Tương tự có biến thành biến thành .

thẳng hàng khi và chỉ khi thẳng hàng. Do là hình chiếu vuông góc của lên các đường nên theo định lí Simson thẳng hàng nằm trên đường tròn ngoại tiếp

  1. [ HỘI CÁC TRƯỜNG CHUYÊN VÙNG DUYÊN HẢI VÀ ĐỒNG BẰNG BẮC BỘ. TRƯỜNG THPT CHUYÊN NGUYỄN BỈNH KHIÊM ĐỀ THI MÔN TOÁN KHỐI 11NĂM 2015- TỈNH QUẢNG NAM ]

Cho nhọn có . Hai đường phân giác trong và ngoài của lần lượt cắt đường thẳng tại và ; hai đường phân giác trong và ngoài của lần lượt cắt đường thẳng tại . Giả sử hai đường tròn đường kính và gặp nhau tại một điểm nằm bên trong . Chứng minh rằng .

Hướng dẫn giải

Gọi là trung điểm của đoạn thẳng .

Khi đó hai điểm và nằm trên đường tròn tâm bán kính .

Vì nên ,

Suy ra .

Từ đó .

Do đó Như vậy suy ra

Tương tự ta cũng có .

Ngoài ra

Từ ta đi đến .

Suy ra .

  1. [ TRẠI HÈ HÙNG VƯƠNG LẦN THỨ X, NĂM HỌC 2013-2014. TRƯỜNG THPT CHUYÊN HOÀNG VĂN THỤ -HÒA BÌNH ]

Cho tam giác nội tiếp đường tròn tâm . Đường tròn tâm tiếp xúc với hai cạnh lần lượt tại và tiếp xúc trong với đường tròn tâm tại điểm . Một đường thẳng song song với tiếp xúc với đường tròn tâm tại điểm nằm trong tam giác ABC.

a) Gọi lần lượt là giao điểm thứ hai của và với . Chứng minh rằng song song với

b) Chứng minh rằng: .

Hướng dẫn giải

a) Gọi là giao điểm thức hai của đường thẳng PC với đường tròn tâm , và là giao điểm thứ hai của đường tròn tâm với .

Xét phép vị tự tâm biến đường tròn tâm thành đường tròn tâm , ta có phép vị tự biến lần lượt thành .

Theo tính chất của phép vị tự ta có EF.

Ta có là ảnh của qua , dẫn đến mà , suy ra là điểm chính giữa của cung . Chứng minh tương tự ta có là điểm chính giữa của cung , là điểm chính giữa của cung .

b) Ta có

(tính chất phép vị tự).

(góc tạo bởi tiếp tuyến và dây cung chắn hai cung bằng nhau) và .

Lại có theo tính chất của hai tiếp tuyến kẻ từ một điểm.

Suy ra , dẫn đến . Từ đó ta có điều phải chứng minh

  1. [ THI HSG VĨNH PHÚC NĂM 2008-2009]

Cho tứ giác lồi nội tiếp trong đường tròn tâm . Gọi là giao điểm của với . CMR nếu ba trung điểm của thẳng hàng thì hoặc

PHẦN HÌNH HỌC PHẲNG

LOẠI 1: Chứng minh tính chất: thẳng hàng, đồng quy, song song, vuông góc.

  1. [TRƯỜNG THPT CHUYÊN NGUYỄN TẤT THÀNH TỈNH YÊN BÁI TRẠI HÈ HÙNG VƯƠNG LẦN THỨ X]

Cho tứ giác ABCD nội tiếp đường tròn tâm O. Đường vuông góc với AD tại A cắt BC ở E. Đường vuông góc với AB tại A cắt CD ở F. Chứng minh rằng ba điểm E, O, F thẳng hàng.

Hướng dẫn giải

Ta có cùng bù với góc , các đỉnh A, C lại thuộc hai phía của đường thẳng EF. Lấy K là điểm đối xứng của A qua EF.

Ta có (do t/c đối xứng) suy ra suy ra tứ giác ECKF nội tiếp

Suy ra mà (t/c đối xứng).

mặt khác (cùng phụ ), suy ra mà hai góc này ở hai đỉnh liên tiếp cùng nhìn cạnh DK. suy ra tứ giác ADKC nội tiếp suy ra K thuộc (O).

Vậy EF là đường trung trực của dây AK suy ra E, O, F thẳng hàng.

  1. [TRƯỜNG TRUNG HỌC PHỔ THÔNG CHUYÊN LƯƠNG VĂN TỤY NINH BÌNH]

Cho tam giác ABC nhọn. Gọi D, E, F tương ứng là chân ba đường cao từ A, B, C của tam giác. I và J tương ứng là tâm đường tròn nội tiếp tam giác AEF và BFD. O và O’ là tâm đường tròn ngoại tiếp tam giác AIC và BJD. Chứng minh: OO’ // IJ.

Hướng dẫn giải

Gọi K là tâm đường tròn nội tiếp tam giác CDE.

Ta có: (vì J, K là tâm đường tròn nội tiếp hai tam giác)

Suy ra

Ta có:

Suy ra tứ giác BJKC nội tiếp đường tròn. (1)

Tương tự AIKC nội tiếp đường tròn. (2)

Từ (1) và (2) suy ra

Ta có:

Suy ra

Từ (3) và (4) ta có OO’ // IJ.

  1. [TRƯỜNG THPT CHUYÊN TUYÊN QUANG TRẠI HÈ HÙNG VƯƠNG LẦN THỨ XII]

Cho tam giác ABC nội tiếp đường tròn (O) và ngoại tiếp đường tròn (I). Đường thẳng BI cắt đường tròn (O) tại N khác B, đường thẳng CI cắt đường tròn (O) tại M khác C. Trên cung BC không chứa A của đường tròn (O) lấy điểm G tùy ý (G khác B, C). Gọi J, K lần lượt là tâm các đường tròn nội tiếp các tam giác ABG, ACG. Đường tròn ngoại tiếp tam giác GJK cắt đường tròn (O) tại điểm P khác G. Hai tiếp tuyến của đường tròn (O) tại M, N cắt nhau tại Q. Chứng minh rằng ba điểm P, A, Q thẳng hàng.

Hướng dẫn giải

Ta có G, J, M và G, K, N thẳng hàng. Hai tam giác PJM và PKN có

; .

Suy ra hai tam giác PJM và PKN đồng dạng. Do đó: .

Vì J là tâm đường tròn nội tiếp tam giác ABG và M là điểm chính giữa cung AB của đường tròn ngoại tiếp tam giác ABG nên MJ = MA.

Tương tự NK = NA. Suy ra . Do đó PMAN là tứ giác điều hòa.

Vì PMAN là tứ giác điều hòa nội tiếp đường tròn (O) nên các tiếp tuyến của (O) tại M, N cắt nhau tại điểm Q trên PA hay ba điểm P, A, Q thẳng hàng.

  1. [TRƯỜNG THPT CHUYÊN LÊ HỒNG PHONG NAM ĐỊNH TRẠI HÈ HÙNG VƯƠNG]

Cho tam giác không cân tại nội tiếp đường tròn . Gọi là tâm đường tròn nội tiếp tam giác. Đường tròn tiếp xúc với các cạnh và tiếp xúc trong với tại . Đường tròn tiếp xúc với các cạnh và tiếp xúc trong với tại .

Hướng dẫn giải

1. Gọi lần lượt là tiếp điểm của với các đường tròn và là giao điểm của , . Chứng minh rằng là tiếp tuyến của các đường tròn ngoại tiếp các tam giác .

2. Gọi là giao điểm của và . Chứng minh rằng .

1. Gọi lần lượt là giao điểm thứ 2 của với

Ta có là các tam giác cân tại , mà thẳng hàng nên suy ra . Do đó

Chứng minh tương tự ta có

Mà cùng phía đối với nên suy ra . Suy ra là điểm chính giữa của cung nên thẳng hàng và .

Ta có nên suy ra tứ giác nội tiếp

Chứng minh tương tự ta có tứ giác nội tiếp.

Mặt khác ta có

Vậy là tiếp tuyến của các đường tròn ngoại tiếp các tam giác .

2. Gọi lần lượt là tâm đường tròn ngoại tiếp các tam giác thì tại .

Vì nên tứ giác nội tiếp.

Gọi . Do và là các tam giác cân có các góc ở đỉnh bằng nên chúng đồng dạng. Suy ra . Do đó

do đó thuộc trục đẳng phương của hai đường tròn hay đồng quy tại . Vậy

  1. [TRƯỜNG THPT CHUYÊN LÀO CAI TỈNH LÀO CAI TRẠI HÈ HÙNG VƯƠNG LẦN THỨ XII]

Cho tam giác nội tiếp đường tròn . Trực tâm tam giác là . Gọi là điểm chính giữa cung của đường tròn ngoại tiếp tam giác giao tại , giao tại . Kẻ phân giác trong của góc . Gọi là tâm đường tròn ngoại tiếp tam giác .

a) Chứng minh

b) Chứng minh rằng bán kính đường tròn ngoại tiếp tam giác bằng

Hướng dẫn giải

a) Ta có nên tứ giác nội tiếp.

Suy ra , ta thhu được tứ giác nội tiếp, tương tự tứ giác nội tiếp.

Dễ có (đường tròn qua 3 điểm , gọi là tâm của . Ta có:

Suy ra .

b) Ta có nên và đối xứng với nhau qua .

Do và nên . Suy ra .

Ta có giao tại là tâm của phép vị tự quay biến thành nên . Do và nên .

Ta thu được , suy ra tứ giác nội tiếp. Mà nên là hình thang cân. Vậy , hay .

  1. [TRƯỜNG THPT CHUYÊN CHU VĂN AN TỈNH LẠNG SƠN TRẠI HÈ HÙNG VƯƠNG LẦN THỨ XII ]

Cho tam giác ABD nội tiếp đường tròn (O). Gọi M là trung điểm của BCAM cắt (O) tại D. Gọi E, F, G, H là trung điểm của AB, BD, DC, CA. Phân giác trong các góc cắt EG, FH tương ứng tại S, T. Gọi .

a. Chứng minh rằng .

b. . Chứng minh rằng AD đi qua trung điểm của PR.

Hướng dẫn giải

a)

Ta có nên

.

Hơn nữa, ME, MG, MF, MH là các đường trung tuyến trương ứng nên suy ra Vì MS, MT là phân giác nên .

Chú ý EFGH là hình bình hành nên , theo định lí Thales thì suy ra , suy ra (1).

Mặt khác, áp dụng định lí Ceva cho tam giác với các đoạn AM, CY, BX thì ta được nên suy ra (2).

Từ (1), (2) suy ra .

b)

Gọi L là trung điểm của AD. Thế thì các tứ giác LHMF, LEMG, EFGH là hình bình hành nên suy ra EG, FH, LM đồng quy tại trung điểm K mỗi đoạn thẳng.

Mặt khác, dễ thấy suy ra , tức là MT là phân giác góc .

Tương tự thì MS là phân giác góc nên suy ra . Do đó MR là phân giác ngoài của .

Tương tự thì MP là phân giác ngoài góc . Suy ra . Dễ thấy AK đi qua trung điểm của EH nên cũng đi qua trung điểm của PR.

Nhận xét và bình luận và phát triển bài toán:

+ Ý a) là kết quả của các tỉ số đồng dạng kết hợp với đường phân giác. Cùng với việc chú ý tới định lí Thales và hình bình hành EFGH.

+ Ý b) là một hệ quả kéo theo với việc nhận thấy từ đó suy ra MS, MR là phân giác trong và ngoài của .

+ Ta có thể thu được kết quả rất thú vị sau: Gọi . Khi đó MPQR là hình chữ nhật.

Chứng minh: Dễ thấy với XE, XG là trung tuyến tương ứng, do đó

nên X, M nằm trên đường tròn Apollonius dựng trên E, G với tỉ số . Do đó, nên M, X, S cùng nằm trên đường tròn Apollonius đó. Hơn nữa, dễ thấy suy ra , tức là MT là phân giác góc . Tương tự thì MS là phân giác góc nên suy ra (3). Do đó, MR là phân giác ngoài của và MS là phân giác trong, nên đường tròn đường kính MS là đường tròn Apollonius dựng trên E, G. Do đó, tứ giác MSXR nội tiếp. Gọi I là trùng điểm của MX. Xét tứ giác toàn phần XDMCAB, thì E, G, I thẳng hàng (nằm trên đường thẳng Gauss). Do đó, SR là đường kính đường tròn Apollonius đó. Vậy, (3). Tương tự thì (5). Từ (3), (4), (5) suy ra MPQR là hình chữ nhật.

  1. [TRƯỜNG THPT CHUYÊN NGUYỄN BỈNH KHIÊM QUẢNG NAM NĂM 2014 KỲ THI OLYMPIC TOÁN KHU VỰC DUYÊN HẢI & ĐỒNG BẰNG BẮC BỘ NĂM 2014]

Cho tứ giác ABCD nội tiếp có các cặp cạnh đối không song song. Các đường thẳng AB và CD cắt nhau tại điểm E và các đường chéo AC và BD cắt nhau tại F. Đường tròn ngoại tiếp các tam giác AFD và BFC cắt nhau tại điểm thứ hai K. Chứng minh rằng hai đường thẳng EK và FK vuông góc.

Hướng dẫn giải

▪ Gọi G là giao điểm của AD và BC, O là tâm đường tròn ngoại tiếp tứ giác ABCD.

Ta dùng kí hiệu (ABC), (ABCD) tương ứng để chỉ đường tròn ngoại tiếp của tam giác ABC, tứ giác ABCD.

Ta có AD, BC, FK lần lượt là trục đẳng phương của các cặp đường tròn (ABCD) và (ADF), (ABCD) và (BCF), (ADF) và (BCF) nên AD, BC, FK đồng quy tại G hay F, K, G thẳng hàng.

▪ Không mất tổng quát ta giả sử F nằm giữa K và G.

Ta có

(ta có thể dùng góc định hướng cho mọi trường hợp).

Suy ra các điểm D, C, K, O cùng thuộc một đường tròn ta gọi là (C1).

▪ Tương tự, các điểm A, B, K, O cùng thuộc một đường tròn ta gọi là (C2).

Ta có AB, CD, OK lần lượt là trục đẳng phương của các cặp đường tròn (ABCD) và (C2),

(ABCD) và (C1), (C1) và (C2) nên AB, CD, OK đồng quy tại E hay O, K, E thẳng hàng.

▪ Xét cực và đối cực đối với đường tròn (O), ta có GF là đối cực của E

nên GF vuông góc với OE

Mà G, K, F thẳng hàng; O, K, E thẳng hàng nên EK và FK vuông góc (điều phải chứng minh).

  1. [Trường THPT Chuyên Hưng Yên ĐỀ THI CHỌN HỌC SINH GIỎI MÔN TOÁN LỚP 11VÙNG DUYÊN HẢI VÀ ĐỒNG BẰNG BẮC BỘ]

Cho đường tròntâm vàlà hai đường kính của đường tròn đó. Tiếp tuyến với đường tròntạicắttại Gọilà giao điểm thứ hai của đường thẳng với đường tròn Gọilà trung điểm của Chứng minh rằng

a) Các điểm cùng nằm trên một đường tròn

b) Ba đường thẳng đồng qui.

Hướng dẫn giải

a). Ta có nên Mà suy ra 4 điểm

nằm trên đường tròn đường kính

b) Gọi là trung điểm của Ta có nên

Mà Tam giác vuông tại có suy ra

(c.c.c), do đó

Từ (1),(2),(3) ta có suy ra 4 điểm nằm trên một đường tròn

Ta có

Hơn nữasuy ralà trục đẳng phương của hai đường tròn và

là trục đẳng phương của hai đường tròn và là trục đẳng phương của hai đường tròn và

Vậy ba đường thẳng đồng qui tại S là tâm đẳng phương của ba đường trònvà

  1. [TRƯỜNG THPT CHUYÊN HOÀNG VĂN THỤ HÒA BÌNH ĐỀ THI CHỌN HỌC SINH GIỎI BẬC THPT VÙNG DUYÊN HẢI &ĐỒNG BẰNG BẮC BỘ LẦN THỨ VII, NĂM HỌC 2011-2012]

. Cho tam giác nhọn ABC, các đường cao AA1, BB1, CC1 đồng quy tại trực tâm H. Đường vuông góc kẻ từ H tới đường thẳng B1C1 và A1C1 lần lượt cắt các đường CA và CB tại P và Q. Chứng minh rằng đường vuông góc kẻ từ C tới A1B1 đi qua trung điểm của PQ.

Hướng dẫn giải

1. Cho tam giác nhọn ABC, các đường cao AA1, BB1, CC1 đồng quy tại trực tâm H. Đường vuông góc kẻ từ H tới đường thẳng B1C1 và A1C1 lần lượt cắt các đường CA và CB tại P và Q. Chứng minh rằng đường vuông góc kẻ từ C tới A1B1 đi qua trung điểm của PQ.

Xét phép vị tự V tâm C biến H thành C1. Gọi P1, Q1 lần lượt là ảnh của P, Q qua phép vị tự V tâm C. Từ tính chất của phép vị tự có , .

Gọi N, K, L lần lượt là hình chiếu của C, P1, Q1 trên A1B1. Khi đó ta có KLP1Q1 là hình thang vuông tại K và L, vì CNA1B1 nên ta chỉ cần chứng minh N là trung điểm của KL thì CN trở thành đường trung bình của hình thang và do đó nó đi qua trung điểm của P1Q1, và theo tính chất của phép vị tự thì nó cũng đi qua trung điểm của PQ.

Có là tứ giác nội tiếp .

Chứng minh tương tự có . Suy ra dẫn đến AC là đường phân giác ngoài của .

Chứng minh tương tự có BC là đường phân giác ngoài của . Do đó điểm C trở thành tâm đường tròn bàng tiếp góc của

Từ tính chất của đường phân giác suy ra được .

Chứng minh tương tự có .

Dẫn đến nếu gọi là nửa chu vi thì .

Do C là tâm đường tròn bàng tiếp góc của nên tính được

. Vậy N là trung điểm của KL. (đpcm)

  1. [Tr­êng THPT chuyªn BẮC GIANG tØnh BẮC GIANG TRẠI HÈ HÙNG VƯƠNG IX

MÔN: TOÁN 11 n¨m häc 2012 – 2013]

Cho tam giác ABC ngoại tiếp đường tròn (I) tâm I. Gọi D, E, F là tiếp điểm của (I) với BC, CA, AB. AD cắt (I) tại điểm thứ hai là X, BX cắt (I) tại điểm thứ hai là Y, CX cắt (I) tại điểm thứ hai là Z. Chứng minh rằng BZ, CY, AX đồng quy.

Hướng dẫn giải

Kẻ tiếp tuyến tại X của (I) cắt BC tại K.

Trong tứ giác XEDF ta có tiếp tuyến tại F, E và XD đồng quy tại A nên tứ giác XEDF là tứ giác điều hòa. Mà KX, KD là tiếp tuyến của (I) tại X, D nên

Mặt khác AD, BE, CF đồng quy nên

Suy ra:

(1)

Theo định lí Céva thì BZ, CY, AX đồng quy

(do )

(luôn đúng theo (1))

Vậy BZ, CY, AX đồng quy (đpcm).

  1. [Tr­êng THPT TrÇn Nguyªn H·n VÜnh Phóc n¨m 2009 – 2010]

Cho tam gi¸c ABC c©n tại A. Gọi H là trung điểm của BC, D là h×nh

chiếu vu«ng gãc của H trªn cạnh AC, M là trung điểm của HD.

Chứng minh AM vu«ng gãc với BD.

Hướng dẫn giải

  1. [TRƯỜNG THPT CHUYÊN THÁI NGUYÊN TRẠI HÈ HÙNG VƯƠNG LẦN THỨ XII]

Kí hiệu A1, B1, C1 là trung điểm các cạnh BC, CA, AB của tam giác nhọn ABC. A2, B2, C2 là hình chiếu của A, B, C trên B1C1, C1A1, A1B1. Trung điểm các đoạn A2B2, C2A2, A2B2A3, B3, C3. Chứng minh các đường thẳng đồng quy.

Hướng dẫn giải

Cách 1: Gọi (O) là đường tròn ngoại tiếp tam giác.

Ta có: .

Dựng hình bình hành có:

, .

Do đó .

Vì đi qua trung điểm EF nên đi qua trung điểm đi qua O. Tương tự đi qua O.

Cách 2: Dùng định lí Ceva dạng sin.

Đặt,

Lâp tỉ số: .

.

Tương tự:

đồng quy.

  1. [CHUYÊN LÊ QUÝ ĐÔN QUẢNG TRỊ ĐỀ THI CHỌN HỌC SINH GIỎI ĐỀ THI CHỌN HỌC SINH GIỎI VÙNG DUYÊN HẢI BẮC BỘ LẦN THỨ VII – NĂM 2014]

Cho tứ giác lồi ABCD có các đường chéo AC và BD cắt nhau tại O.

Gọi P, Q là tâm các đường tròn ngoại tiếp tam giác AOB và tam giác COD. Chứng minh rằng AB + CD 4.PQ

Hướng dẫn giải

* Đường phân giác góc cắt đường tròn ngoại tiếp tam giác AOB tại E O.

Tứ giác AEBO nội tiếp =>

Nếu thì EO > EA

Nếu thì EO >EB = EA

Vậy ta luôn có EO > EA

* Đường phân giác góc cắt đường tròn ngoại tiếp tam giác COD tại F O. Tương tự ta có FO > FD

*Gọi I, J là hình chiếu của P, Q lên đường thẳng EF ta có:

AB < AE + EB 2EO

CD < CF + FD 2FO

=> AB + CD < 2EF = 4IJ 4PQ

  1. [TRƯỜNG THPT CHUYÊN HẠ LONG QUẢNG NINH ĐỀ THI OLYMPIC TRẠI HÈ HÙNG VƯƠNG LẦN THỨ X MÔN: TOÁN - KHỐI: 11 Ngày thi: 01 tháng 08 năm 2014]

Cho tam giác ABC nội tiếp đường tròn tâm O. Đường tròn tâm I tiếp xúc với hai cạnh AC, BC lần lượt tại E, F và tiếp xúc trong với đường tròn tâm O tại điểm P. Một đường thẳng song song với AB và tiếp xúc với đường tròn tâm I tại điểm Q nằm trong tam giác ABC.

a) Gọi K, L lần lượt là giao điểm thứ hai của PE và PF với (O). Chứng minh rằng KL song song với EF.

b) Chứng minh rằng .

Hướng dẫn giải

a. Xét phép vị tự tâm P biến đường tròn (I) thành đường tròn (O) nên biến điểm E thành điểm K và biến điểm F thành điểm L nên KL//EF.

b. Gọi D là giao điểm thức hai của đường thẳng PC với đường tròn tâm I, và M là giao điểm thứ hai của đường tròn tâm O với PQ.

Xét phép vị tự biến đường tròn tâm I thành đường tròn tâm O, ta có phép vị tự biến E, D, Q, F lần lượt thành K, C, M, L.

Do OK là ảnh của IE qua , dẫn đến mà nên , suy ra K là điểm chính giữa của cung AC.

Chứng minh tương tự ta có L là điểm chính giữa của cung BC, M là điểm chính giữa của cung AB.

Nếu thì ta có

Trường hợp ta cũng chỉ ra được

(tính chất phép vị tự).

(góc tạo bởi tiếp tuyến và dây cung chắn hai cung bằng nhau) và DE = QF.

Lại có CE = CF theo tính chất của hai tiếp tuyến kẻ từ một điểm.

Suy ra , dẫn đến . Từ đó ta có điều phải chứng minh.

  1. [TRƯỜNG THPT CHUYÊN BẮC GIANG ĐỀ THI CHỌN HỌC SINH GIỎI BẬC THPT VÙNG DUYÊN HẢI ĐỒNG BẰNG BẮC BỘ NĂM HỌC 2012 – 2013]

Cho tam giác ABC. Đường tròn (I) tâm I nội tiếp tam giác ABC, tiếp xúc với BC, CA, AB lần lượt tại P, Q, R. Một đường tròn đi qua B, C tiếp xúc với (I) tại X. Một đường tròn đi qua C, A tiếp xúc với (I) tại Y. Một đường tròn đi qua A, B tiếp xúc với (I) tại Z. Chứng minh rằng các đường thẳng PX, QY, RZ đồng quy.

Hướng dẫn giải

  1. [Ngân hàng đề Trường THPT Chuyên Hạ Long - Sở GD- ĐT Quảng Ninh]

Giả sử đường tròn (I) nội tiếp tam giác ABC tiếp xúc với các cạnh BC, CA, AB theo thứ tự D, E, F. Đường thẳng qua A và song song với BC cắt EF tại K. Gọi M là trung điểm của BC. Chứng minh rằng: IM DK.

Hướng dẫn giải

Gọi N là giao điểm của ID và EF. Qua N kẻ đường thẳng // BC cắt AB, AC theo thứ tự từ P,Q. Vì hai tứ giác IFPN và IQEN nội tiếp nên

1.0 đ

Mặt khác . Do đó cân tại I. Vậy N là trung điểm của PQ -> A, N, M thẳng hàng. 0.5 đ

Lại có -> N là trực tâm 0.5 đ

Gọi H là giao điểm của AM và IK

J là giao điểm của IA và EF

1.0 đ

Mà nội tiếp nên(Đpcm). 1.0đ

  1. [TRẠI HÈ HÙNG VƯƠNG LẦN THỨ XI-TRƯỜNG THPT CHUYÊN TUYÊN QUANG -LỚP 11]

Cho tam giác ABC với AB > AC. Các đường trung tuyến và phân giác trong góc A cắt BC tại MN tương ứng. Đường thẳng qua N vuông góc với AN cắt AB, AM lần lượt tại PQ; đường thẳng qua P vuông góc với AB cắt đường thẳng AN tại R. Chứng minh QR vuông góc với BC.

Hướng dẫn giải

Gọi D là giao điểm thứ hai của AN với đường tròn ngoại tiếp tam giác ABC, dễ thấy suy ra DM vuông góc với BC.

Đặt và xét phép vị tự . Khi đó B’ thuộc AB, C’ thuộc AC và hai tam giác BCDB’C’R có các cạnh tương ứng song song.

Gọi K là giao điểm của PN với B’C’, ta có

suy ra tứ giác RKPB’ nội tiếp. Từ đó

Như vậy nên K là trung điểm B’C’, hay K thuộc AM, suy ra K trùng Q. Do B’C’ song song với BCQR vuông góc với B’C’ nên QR vuông góc với BC.

  1. [TRẠI HÈ HÙNG VƯƠNG LẦN THỨ XI-ĐỀ CHÍNH THỨC -LỚP 11]

Cho tam giác có ba góc nhọn, và nội tiếp đường tròn Các đường cao cắt nhau tại Đường thẳng cắt tại Lấy điểm trên sao cho Đường tròn ngoại tiếp tam giác cắt tại Chứng minh rằng

a) Ba điểm thẳng hàng.

b) Đường thẳng vuông góc với đường thẳng

Hướng dẫn giải

  1. [TRƯỜNG THPT CHUYÊN LÀO CAI ĐỀ THI ĐỀ XUẤT KỲ THI HSG VÙNG DUYÊN HẢI VÀ ĐỒNG BẰNG BẮC BỘ LẦN THỨ VII MÔN TOÁN: KHỐI 11 Năm học: 2013-2014]

Cho tam giác nhọn ABC không cân. Gọi H, O lần lượt là trực tâm, tâm đường tròn ngoại tiếp tam giác ABC; D, E lần lượt là chân đường cao hạ từ các đỉnh A, B của tam giác ABC. Các đường thẳng ODBE cắt nhau tại K, các đường thẳng OEAD cắt nhau tại L. Gọi M là trung điểm cạnh AB. Chứng minh rằng ba điểm K, L, M thẳng hàng khi và chỉ khi bốn điểm C, D, O, H cùng nằm trên một đường tròn.

Hướng dẫn giải

Áp dụng đinh lý Mê-nê-la-uýt cho tam giác HAB và ba điểm K, L, M ta có: K, L, M thẳng hàng khi và chỉ khi (1)

Ta lại có (cùng cạnh đáy OD), (cùng cạnh đáy OE) và .(Bởi vì ,

ở đó R là bán kính đường tròn ngoại tiếp tam giác ABC và . Tương tự ).

Từ các kết quả trên ta có khi và chỉ khi OH // DE hoặc OH đi qua trung điểm ED.

Bằng cách vẽ tiếp tuyến của đường tròn ngoại tiếp tam giác ABC tại C, dễ dàng suy ra DE // , suy ra CO vuông góc với DE.

Gọi P, Q lần lượt là trung điểm của DE, HC. Dễ thấy tứ giác CEHD nội tiếp, suy ra QP vuông góc với DE. Suy ra CO//QP.

Nếu HO đi qua trung điểm DE suy ra P là trung điểm HO, suy ra EHDO là hình bình hành, suy ra OD // EHEO // HD. Điều này trái với giả thiết OD cắt BEOE cắt AD.

Vậy (1) xảy ra khi và chỉ khi OH // DE khi và chỉ khi CO vuông góc với OH khi và chỉ khi E, H, O, D cùng nằm trên một đường tròn (vì ta luôn có tứ giác CEHD nội tiếp đường tròn đường kính CH).

  1. [TRƯỜNG THPT CHUYÊN VĨNH PHÚC ĐỀ THI CHỌN HỌC SINH GIỎI VÙNG DUYÊN HẢI VÀ ĐỒNG BẰNG BẮC BỘ LẦN THỨ VIII- NĂM 2015 MÔN TOÁN - LỚP 11]

Cho tam giác nhọn không cân, nội tiếp đường tròn . là điểm nằm trong tam giác sao cho . Đường tròn đường kính cắt các cạnh lần lượt tại và cắt đường tròn tại điểm khác . Chứng minh rằng đồng quy.

Hướng dẫn giải

Gọi AD là đường kính của (O), dễ thấy G,P,D thẳng hàng và . Giả sử PE,PF cắt DB,DC tại K,L; EF cắt BC tại T.

Theo định lý Desargues để chứng minh BE, CF, GP (hay PD) đồng quy ta chỉ cần chứng minh T,K,L thẳng hàng.

Áp dụng định lý Menelaus ta được: (1)

Dễ thấy tứ giác EFBC nội tiếp nên (2)

Cũng từ EFBC nội tiếp suy ra

Tứ giác PKDL là hình bình hành suy ra .

Suy ra (3).

Ta có (4)

Thay (2), (3), (4) vào (1) ta được . Từ đó áp dụng định lý menelaus cho tam giác DBC ta suy ra T,K,L thẳng hàng. Bài toán được chứng minh.

  1. [TRƯỜNG THPT CHUYÊN BẢO LỘC- LÂM ĐỒNG- KỲ THI HSG KHU VỰC DH VÀ ĐBBB LẦN THỨ 9 ĐỀ THI ĐỀ NGHỊ MÔN: TOÁN; LỚP: 11]

Cho tứ giác ABCD nội tiếp có các cặp cạnh đối không song song. Các đường thẳng AB và CD cắt nhau tại điểm E và các đường chéo AC và BD cắt nhau tại F. Đường tròn ngoại tiếp các tam giác AFD và BFC cắt nhau tại điểm thứ hai K. Chứng minh rằng hai đường thẳng EK và FK vuông góc.

Hướng dẫn giải

▪ Gọi G là giao điểm của AD và BC, O là tâm đường tròn ngoại tiếp tứ giác ABCD.

Ta dùng kí hiệu (ABC), (ABCD) tương ứng để chỉ đường tròn ngoại tiếp của tam giác ABC, tứ giác ABCD.

Ta có AD, BC, FK lần lượt là trục đẳng phương của các cặp đường tròn (ABCD) và (ADF), (ABCD) và (BCF), (ADF) và (BCF) nên AD, BC, FK đồng quy tại G hay F, K, G thẳng hàng.

▪ Không mất tổng quát ta giả sử F nằm giữa K và G.

Ta có

(ta có thể dùng góc định hướng cho mọi trường hợp).

Suy ra các điểm D, C, K, O cùng thuộc một đường tròn ta gọi là (C1).

▪ Tương tự, các điểm A, B, K, O cùng thuộc một đường tròn ta gọi là (C2).

Ta có AB, CD, OK lần lượt là trục đẳng phương của các cặp đường tròn (ABCD) và (C2),

(ABCD) và (C1), (C1) và (C2) nên AB, CD, OK đồng quy tại E hay O, K, E thẳng hàng.

▪ Xét cực và đối cực đối với đường tròn (O), ta có GF là đối cực của E

nên GF vuông góc với OE

Mà G, K, F thẳng hàng; O, K, E thẳng hàng nên EK và FK vuông góc (điều phải chứng minh).

  1. [KỲ THI HỌC SINH GIỎI CÁC TRƯỜNG THPT CHUYÊN KHU VỰC DUYÊN HẢI VÀ ĐỒNG BẰNG BẮC BỘ LẦN THỨ IX, NĂM HỌC 2015 – 2016 ĐỀ THI MÔN TOÁN – KHỐI 11]

Cho tam giác ABC nội tiếp đường tròn (O). Tiếp tuyến của (O) tại B, C cắt nhau tại S. Gọi d là đường thẳng chứa phân giác trong góc A của tam giác ABC. Các đường trung trực của các đoạn thẳng AB, AC cắt d lần lượt tại M và N. Gọi P là giao điểm của BM và CN, I là tâm đường tròn nội tiếp tam giác MNP, H là trực tâm của tam giác OMN.

a. Chứng minh H, I đối xứng với nhau qua d.

b. Chứng minh A, I, S thẳng hàng.

Hướng dẫn giải

+) Chứng minh OP là trung trực của MN

Không mất tính tổng quát ta giả sử bài toán có vị trí tương đối như hình vẽ.

Gọi D là trung điểm của BC, E là giao điểm (khác A) của d với (O), F là trung điểm của MN.

Vì hai tam giác MAB và NAC cân nên dễ thấy:

Suy ra, tam giác PMN cân tại P và tam giác OMN cân tại O. Vậy OP là trung trực của MN.

+) Chứng minh I, H đối xứng với nhau qua d

Ta có:

Vậy hai điểm I và H đối xứng với nhau qua d.

+) Chứng minh AD, AS đối xứng với nhau qua AE

Gọi EK là đường kính của (O).

Ta có (DSEK) = - 1 nên A (DSEK) = -1 mà AE và AK vuông góc với

nhau suy ra AE là phân giác góc SAD.

Vậy AD, AS đối xứng với nhau qua AE.

+) Dựa vào tính chất của phép đối xứng trục d ta thấy A, I, S thẳng hàng khi và chỉ khi A, H, D thẳng hàng. Ta dùng Melenauyt với tam giác OEF để chứng minh điều này.

Ta có điều phải chứng minh

  1. [TRƯỜNG THPT CHUYÊN BIÊN HÒA HỘI CÁC TRƯỜNG THPT CHUYÊN VÙNG DUYÊN HẢI VÀ ĐỒNG BẰNG BẮC BỘ]

Cho tam giác ABC với H là trực tâm tam giác, O là tâm đường tròn ngoại tiếp và R là bán kính đường tròn ngoại tiếp. Gọi D là điểm đối xứng của A qua BC, E là điểm đối xứng của B qua CA, F là điểm đối xứng của C qua AB. Chứng minh rằng D, E, F thẳng hàng khi và chỉ khi OH = 2R.

Hướng dẫn giải

Gọi G là trọng tâm tam giác ABC. lần lượt là trung điểm BC, CA, AB. Gọi I, J,K là tam giác nhận, A, B, C là trung điểm các cạnh JK, KI, IJ. Do đó G là trọng tâm tam giác IJK.

Từ cách dựng suy ra HA, HB, HC lần lượt là đường trung trực của JK, KI, IJ. Do đó H là tâm đường tròn ngoại tiếp tam giác IJK có bán kinh 2R.

Gọi là hình chiếu vuông góc của O lên các đường JK, KI, IJ. Do G là trọng tâm hai tam giác trên nên:

Xét V(G; -1/2) biến A, B, C, I, J, K thành

Có là trung điểm BC nên ∟BC, ∟BC. Vậy thẳng hàng. Do đó vuông góc với BC.

Suy ra // AD và

Vậy Thẳng hàng và biến D thành

Tương tự có biến E thành biến F thành

D, E, F thẳng hàng khi và chỉ khi thẳng hàng. Do là hình chiếu vuông góc của O lên các đường JK, KI, IJ nên theo định lí Simson thẳng hàng O nằm trên đường tròn ngoại tiếp

  1. Cho tam giác có ba góc nhọn, và nội tiếp đường tròn Các đường cao cắt nhau tại Đường thẳng cắt tại Lấy điểm trên sao cho Đường tròn ngoại tiếp tam giác cắt tại Chứng minh rằng

a) Ba điểm thẳng hàng.

b) Đường thẳng vuông góc với đường thẳng

a) Dễ thấy tứ giác nội tiếp.

Hướng dẫn giải

Hơn nữa,

nên ngũ giác nội tiếp.

Do đó là ba trục đẳng phương của và nên chúng

đồng quy tại tức là ba điểm thẳng hàng.

b) Nối cắt tại suy ra thẳng hàng.

Dễ thấy

Từ đó suy ra Do đó (1).

Hơn nữa, (2). Từ (1) và (2) suy ra . Mà nên là trung trực của suy ra .

  1. Tứ giác lồi diện tích và không có hai cạnh nào song song.Lấy điểm nằm trên đường thẳng sao cho và nằm cùng phía so với đường thẳng và .Tương tự cũng có các điểm lần lượt nằm trên các đường thẳng và .Chứng minh thẳng hàng.

Hướng dẫn giải

Gọi x là khoảng cách từ D đến AC và y là khoảng cách từ B đến AC.Ta có:

Lấy Q1 nằm trên đường thẳng BC sao cho .Khi đó

,khi đó với mọi điểm O ta có (1)

Tương tự ta cũng có các đẳng thức dưới đây:

(2)

(3)

(4)

Khi đó:

(từ (1) và (2) )

( từ (3) và (4) )

Suy ra

Hoàn toàn tương tự ta cũng có: và .Suy ra P1,P2,P3,P4 thẳng hàng.

  1. Cho là trực tâm tam giác không cân góc nhọn. Hình chiếu vuông góc của trên theo thứ tự là Gọi là trung điểm là giao điểm của hai đường tròn đường kính và . Chứng minh thẳng hàng và các đường thẳng đồng quy.

Hướng dẫn giải

Gọi G là chân đường cao kẻ từ đỉnh A của tam giác ABC.

Ta có suy ra H nằm trên trục đẳng phương của hai đường tròn đường kính BC và AD. Suy ra H, P, Q thẳng hàng.

Gọi I là giao điểm của EF và BC. (DEF) là đường tròn Euler của tam giác ABC nên G nằm trên (DEF). Do đó

Suy ra I, P, G thẳng hàng hay BC, EF, PQ đồng quy tại I.

  1. Cho tam giác với là trực tâm tam giác, là tâm đường tròn ngoại tiếp và là bán kính đường tròn ngoại tiếp. Gọi là điểm đối xứng của qua , là điểm đối xứng của qua , là điểm đối xứng của qua . Chứng minh rằng thẳng hàng khi và chỉ khi .

Hướng dẫn giải

Gọi G là trọng tâm tam giác ABC. lần lượt là trung điểm BC, CA, AB. Gọi I, J,K là tam giác nhận, A, B, C là trung điểm các cạnh JK, KI, IJ. Do đó G là trọng tâm tam giác IJK.

Từ cách dựng suy ra HA, HB, HC lần lượt là đường trung trực của JK, KI, IJ. Do đó H là tâm đường tròn ngoại tiếp tam giác IJK có bán kinh 2R.

Gọi là hình chiếu vuông góc của O lên các đường JK, KI, IJ. Do G là trọng tâm hai tam giác trên nên:

Xét V(G; -1/2) biến A, B, C, I, J, K thành

Có là trung điểm BC nên ∟BC, ∟BC. Vậy thẳng hàng. Do đó vuông góc với BC.

Suy ra // AD và

Vậy Thẳng hàng và biến D thành

Tương tự có biến E thành biến F thành

D, E, F thẳng hàng khi và chỉ khi thẳng hàng. Do là hình chiếu vuông góc của O lên các đường JK, KI, IJ nên theo định lí Simson thẳng hàng O nằm trên đường tròn ngoại tiếp

  1. Cho hai đường tròn và cắt nhau tại hai điểm và là đường kính của đường tròn . Vẽ tiếp tuyến của đường tròn tại điểm cắt đường tròn tại điểm thứ hai là . Đường thẳng cắt đường tròn tại , . Đường thẳng cắt đường tròn tại . Giả sử là một điểm bất kì trên đoạn thẳng . Đường thẳng cắt đường tròn tại và đường thẳng cắt đường thẳng tại . Chứng minh rằng .

Hướng dẫn giải

Vì là đường kính của đường tròn và là tiếp tuyến nên . Do đó là đường kính của .

Ta lại có .

Nối , ta có . Suy ra

Xét tam giác và tam giác có:

+

+

Suy ra hai tam giác và đồng dạng. Do đó

Mặt khác là tiếp tuyến của đường tròn nên .

Suy ra

Vì vậy

  1. Cho tam giác nhọn không cân. Gọi lần lượt là trực tâm, tâm đường tròn ngoại tiếp tam giác hai đường cao . cắt tại , cắt tại . Gọi là trung điểm . Chứng minh rằng thẳng hàng khi và chỉ khi bốn điểm cùng nằm trên một đường tròn.

Hướng dẫn giải

Sử dụng Menelaus cho tam giác HAB và 3 điểm K, L, M thẳng hàng

Vậy

Do đó hệ thức xảy ra khi và chỉ khi hoặc OH đi qua trung điểm P của DE.

Qua C kẻ tiếp tuyến d với đường tròn (C) thì d song song với DE.

Do CO vuông góc với d nên CO vuông góc với DE.

Nếu OH đi qua P thì P là trung điểm của OH, hay EDOH là hình bình hành, suy ra EO và HD song song (trái giả thiết).

Vậy K, L,M thẳng hàng khi và chỉ khi OH song song với DE, hay OH vuông góc với CO, tương đương C, D, O, H cùng thuộc đường tròn đường kính CH.

  1. Cho tam giác . Gọi là điểm thỏa mãn và giả sử . Gọi là hình chiếu vuông góc của trên là trung điểm của đoạn . Chứng minh rằng vuông góc với

Hướng dẫn giải

Gọi D là điểm đối xứng với B qua CK. Khi đó . Do nên Vì

=nên

⇒⇒

⇒ và tứ giác CHDA là tứ giác nội tiếp⇒

Do AH và BD cùng vuông góc với CK nên AH // DB⇒

Từ (1) và (2) suy ra tứ giác CEFD là tứ giác nội tiếp (DH cắt BC tại E, CK cắt BC tại F) suy ra H là trực tâm tam giác BCD ⇒ BH ⊥ CD⇒ BH//AD

Vậy tứ giác AHBD là hình bình hành, do M là trung điểm của AB nên H, M, D thẳng hang. Vậy MH ⊥ BC.

Cho hai đường tròn và ngoài nhau. Kẻ tiếp tuyến chung ngoài , tiếp tuyến chung trong của hai đường tròn (, . Chứng minh rằng ba đường thẳng đồng quy.

  1. Cho tam giác nhọn với nội tiếp đường tròn . Gọi là trung điểm của cạnh và là hình chiếu của trên . Gọi là đường tròn đi qua và tiếp xúc với tại điểm khác A.

a) Gọi là giao điểm thứ hai của đường thẳng với . Chứng minh rằng là hình thang cân.

b) Chứng minh rằng đường thẳng đi qua trọng tâm của tam giác .

Hướng dẫn giải

Gọi và là tiếp tuyến tại AX của đường tròn và gọi là đường tròn ngoại tiếp tam giác . Dễ thấy a cũng là tiếp tuyến của tại A và a là trục đẳng phương của hai đường tròn và .

Như vậy ba đường thẳng và lần lượt là trục đẳng phương của các cặp đường tròn và ; và ; và , do đó và đồng quy tại điểm M.

Ta có nên M là tâm đường tròn ngoại tiếp tam giác ADX. Chú ý là . Ta có

, suy ra . Do đó là hình thang cân.

b) (2,0 điểm)

Gọi là trung điểm của . Xét phép vị tự biến ; ; ; , suy ra .

Mặt khác . Do đó , từ đó suy ra thẳng hàng và ta có điều phải chứng minh.

Chú ý: Muốn cho bài toán khó lên thì có thể chỉ hỏi phần (b).

  1. Cho tam giác nhọn nội tiếp đường tròn tâm , là chân đường cao xuất phát từ và là trực tâm của tam giác Đường thẳng qua vuông góc với cắt cạnh tại Chứng minh rằng

Gọi là giao điểm của và là giao điểm của và .

Theo định lý con bướm, suy ra là trung điểm của đoạn

Mặt khác là trung điểm của , do đó tứ giác là một hình bình hành. Suy ra

Mà do vậy

  1. Cho tam giác nhọn nội tiếp đường tròn có trực tâm là và là một điểm thay đổi trên cung nhỏ . là điểm đối xứng của qua trung điểm của .

a) Chứng minh rằng trực tâm của tam giác nằm trên đường tròn .

b) Giả sử cắt tại , hạ vuông góc với tại . Chứng minh rằng ba điểm và trung điểm của thẳng hàng.

Hướng dẫn giải

a) Chứng minh K thuộc đường tròn (O)

+ N là điểm đối xứng của M qua trung điểm I của AB nên tứ giác ANBM là hình bình hành, suy ra và

+ Vì K là trực tâm tam giác NAB nên ,

Do đó và

Từ đó suy ra tứ giác BKAM nội tiếp.

Vậy K thuộc đường tròn (O).

b) Chứng minh rằng DE đi qua trung điểm của HK.

+ Gọi S là điểm đối xứng của K qua E; R là điểm đối xứng của K qua D. Ta có: (do BC là đường trung trực của SK)

+ Mặt khác (cùng chắn cung BC) nên .

+ Mà nên

Suy ra tứ giác BHCS nội tiếp nên

+ Tương tự tứ giác ABHR nội tiếp nên

+ Từ (1) và (2) ta có

Suy ra S, H, R thẳng hàng.

+ Vì DE là đường trung bình của tam giác KRS, nên DE đi qua trung điểm của HK.

  1. (Kỳ thi HSG trường THPT chuyên Trần Phú – Hải Phòng năm học 2014 – 2015) Hai đường tròn tiếp xúc ngoài với nhau tại và cùng tiếp xúc trong với đường tròn tại tương ứng sao cho không thẳng hàng. là tiếp tuyến chung tại của và . là đường kính của sao cho vuông góc với và cùng phía so với . Chứng minh rằng và đồng quy.
  2. (Đề thi đề xuất – trường THPT Chuyên Biên Hòa tỉnh Hà Nam – năm 2015) Cho là tam giác nhọn với đường tròn nội tiếp . Gọi là điểm của đường tròn bang tiếp góc với . Gọi là giao điểm của với ( nằm giữa và . Giả sử cắt đường cao của tại .

a. Chứng minh .

b. Gọi là đường tròn có tâm nằm trên đường cao đi qua và tiếp xúc với đường tròn tại . Các điểm xác định tương tự. Chứng minh đồng quy tại 1 điểm.

Hướng dẫn giải:

a. Gọi là tiếp điểm của với .

Giả sử cắt tại điểm thứ 2 là . Qua vẽ đường thẳng song song với cắt tại điểm .

Ta có: phép vị tự ;

. Do đó thẳng hàng

Khi đó (cùng vuông góc với )

Mà nên suy ra

b. Từ câu a ta suy ra: đường tròn có tâm thuộc đường cao , đi qua và tiếp xúc với tại thì . Do đó .

Tương tự nếu gọi lần lượt là tiếp điểm của đường tròn bang tiếp góc của với thì đồng quy đồng quy.

Mặt khác nếu ta gọi là độ dài 3 cạnh của và là nửa chu vi thì ta có:

Theo định lý Ceva ta có đồng quy.

  1. (Đề thi chọn HSG trường THPT chuyên Thái Bình – 2015) Cho nội tiếp đường tròn . Tiếp tuyến của tại cắt nhau tại . Trung trực của cắt là phân giác trong góc của thứ tự tại và . Gọi là giao của và , là tâm đường tròn nội tiếp của tam giác .

a. Chứng minh đối xứng nhau qua với là trực tâm của .

b. Chứng minh thẳng hàng.

Hướng dẫn giải:

Không mất tính tổng quát ta giả sử bài toán có vị trí tương đối như hình vẽ.

Gọi D là trung điểm của BC, E là giao của phân giác góc A với (O) khác A. F là trung điểm của MN.

a) +) Chứng minh OP là trung trực của MN

Vì hai tam giác cân MABNAC có các cặp góc tương ứng bằng nhau nên ta có: . Suy ra tam giác và cân tại P O. Vậy OP là trung trực của MN.

+) Chứng minh I, H đối xứng nhau qua d

Ta có:

Ta được đpcm.

b)+) Chứng minh AD, AS đối xứng nhau qua AE

Gọi EK là đường kính của (O). Ta có nên mà AEAK vuông góc với nhau suy ra là phân giác . Ta có đpcm

Dựa vào tính chất của phép đối xứng trục ta thấy A, I, S thẳng hàng khi và chỉ khi A, H, D thẳng hàng. Ta dùng định lý Menelaus với tam giác OEF để chứng minh điều này.

đpcm

  1. (Đề thi đề xuất thi HSG trường THPT Chu Văn An – Hà Nội – 2015) nhọn, nội tiếp đường tròn . Kẻ đường kính . thuộc thỏa mãn . cắt tại khác . Chứng minh thẳng hàng, với là trực tâm .

Hướng dẫn giải:

DP cắt AB tại E thì M là trung điểm DE (vì OM là đường trung bình).

BHCD là hình bình hành nên DH cắt DC tại I là trung điểm mỗi đường.

Suy ra MI là đường trung bình của

Kéo dài CH cắt (O) tại Q. Ta sẽ chứng minh bằng cách chứng minh Q, E, D thẳng hàng.

nên BDCQ là hình thang cân (hình thang nội tiếp).

Ta có vì tam giác QBH cân tại B

vì hình thang BDCQ cân

Nên . Mà Q, H, C thẳng hàng nên thẳng hàng hay .

  1. (Đề thi đề xuất trường THPT chuyên Hưng Yên tỉnh Hưng Yên – 2015)

Cho nội tiếp đường tròn . Đường phân giác của góc cắt tại khác . Gọi là điểm đối xứng với qua . cắt tại khác . là một điểm thay đổi trên cạnh . Đường thẳng cắt tại khác . Từ kẻ đường thẳng song song với cắt tại P. Đường tròn ngoại tiếp cắt tại và cắt tại . Chứng minh thẳng hàng và đường thẳng luôn đi qua điểm cố định khi I thay đổi.

Hướng dẫn giải:

Gọi D là điểm chính giữa của cung và E đối xứng với B qua AD nên .

A

M

J

F

B

K

D

C

E

I

P

Q

DF là trung trực của EC.

(do )

Mà (do )

P, Q cùng phía đối với nên thẳng hàng.

Đường thẳng IK cắt (O) tại M khác K

do AD là trung trực của BE

DM là đường kính của (O)

D là điểm chính giữa cung chứa A thì M là điểm chính giữa cung không chứa A nên M cố định. Vậy đường thẳng KI luôn qua điểm M cố định khi I thay đổi.

  1. (Đề thi đề xuất trường THPT chuyên tình Lào Cai – trại hè Hùng Vương lần thứ X)

Cho đường tròn và hai đường kính , . Tiếp tuyến với đường tròn tại cắt tại , cắt đường tròn lần nữa tại . Gọi là giao điểm của với . Chứng minh 3 điểm thẳng hàng.

  1. (Đề thi đề xuất HSG Vùng duyên hải và đồng bằng Bắc Bộ lần thứ VI - trường THPT chuyên Lào Cai) Cho có trực tâm , ba đường cao , , . Gọi , , lần lượt là trung điểm , , . Gọi là đường tròn ngoại tiếp (còn gọi là đường tròn Euler của ). Kí hiệu , , là các giao điểm thứ hai của , , và . Chứng minh rằng , , đồng quy tại một điểm nằm trên đường thẳng đi qua trọng tâm và trực tâm (còn gọi là đường thẳng Euler của ).

Hướng dẫn giải:

Ta kí hiệu đường tròn qua 3 điểm T, U, V là (TUV).

Gọi O là tâm đường tròn ngoại tiếp tam giác M, N, P. Ta biết rằng (W) đi qua 9 điểm: M, N, P, và trung điểm AH, BH, CH. Giả sử M’, N’, P’ là điểm đối xứng với M, N, P qua O.

Xét phép nghịch đảo cực H và giữ bất biến (W). Phép nghịch đảo này biến tương ứng thành các đường tròn .

Ta sẽ chỉ ra rằng trục đẳng phương của (HNN’) và (HPP’) là đường thẳng Euler của tam giác ABC (do đường thẳng này bất biến qua phép nghịch đảo nói trên). Thật vậy:

Trục đẳng phương của (W) và (HNN’) là NN’;

Trục đẳng phương của (W) và (HPP’) là PP’;

Do đó trục đẳng phương của (HNN’) và (HPP’) đi qua H và giao của NN’PP’. Nhưng ta biết rằng tâm O của (W) cũng nằm trên đường thẳng Euler của tam giác ABC. Do đó trục đẳng phương của (HNN’) và (HPP’) chính là đường thẳng Euler của tam giác ABC.

Tương tự, trục đẳng phương của (HPP’) và (HMM’), trục đẳng phương của (HMM’) và (HNN’) cũng là đường thẳng Euler của tam giác ABC.

Do đó ba đường tròn cùng đi qua một điểm trên đường thẳng Euler của tam giác ABC. Từ đó ta có điểu phải chứng minh.

  1. (Đề đề nghị chọn HSG khu vực duyên hải đồng bằng Bắc Bộ 2015 – trường THPT chuyên Bắc Ninh) Cho nhọn với . Giả sử và là các điểm trên cạnh sao cho và nằm giữa và . Giả sử là điểm thuộc miền trong của sao cho và . Chứng minh rằng: .

Hướng dẫn giải:

Vẽ hình bình hành BPCQ, khi đó PQBC giao nhau tại trung điểm M của mỗi đường.

Do đó DEPQ cũng giao nhau tại trung điểm M của mỗi đường suy ra PDQE là hình bình hành. Suy ra từ đó A, E, Q thẳng hàng.

Vẽ hình bình hành BPAT. Khi đó ta cũng suy ra TACQ là hinh bình hành.

Ta có .

Do đó tứ giác TAQB nội tiếp.

Ta thấy qua phép tịnh tiến véc tơ thì tam giác BQT biến thành tam giác PCA.

Do đó (đpcm)

  1. (Đề thi đề xuất trường THPT chuyên tỉnh Thái Nguyên, trại hè Hùng Vương lần thứ 10) Cho tam giác ABC nhọn () có AH là đường cao. P, Q là chân các đường vuông góc hạ từ H xuống AB, AC. Gọi M là giao của PQBC, K là giao của đường tròn ngoại tiếp tam giác ABCAM, I là tâm đường tròn ngoại tiếp tam giác BPC. Chứng minh K, H, I thẳng hàng.

Hướng dẫn giải:

Dễ dàng chứng minh được PQCB là tứ giác nội tiếp.

Ta có MB.MC = MP.MQ. Do hai tam giác MHPMQH đồng dạng nên .

Vậy có suy ra AKPH là tứ giác nội tiếp. Vậy

Ta có năm điểm A, K, P, H, Q cùng thuộc đường tròn tâm J là trung điểm của AH. Bây giờ ta lại có I là tâm đường tròn ngoại tiếp tứ giác PQCB nên . Gọi O là tâm đường tròn ngoại tiếp tam giác ABC ta dễ dàng chứng minh được . Từ đó OA // IJ.

Lại có . Từ đó AJIO là hình bình hành. Dễ dàng suy ra JHIO cũng là hình bình hành. Mà JO vuông góc với AK (do AK là trục đẳng phương của hai đường tròn (J) và (O)). Vậy HI cũng vuông góc với AK. Lại có KH vuông góc với AK nên K, H, I thẳng hàng.

  1. (Đề thi đề xuất trường THPT chuyên tỉnh Sơn La, trại hè Hùng Vương lần thứ XII) Cho ngoại tiếp đường tròn tâm . Các cạnh lần lượt tiếp xúc với đường tròn tại . Gọi lần lượt là các trung điểm các cạnh và , là giao điểm của các đường thẳng và .

a. Chứng minh rằng các điểm thẳng hàng.

b. Gọi là điểm thỏa và . Chứng minh .

Hướng dẫn giải:

a. Kéo dài AP cắt CB tại S. Vì M, K là các trung điểm ACAB nên P là trung điểm AS.

+ Trong tam giác CASCP là trung tuyến và phân giác nên

+ Đặt . Có (1)

+

.

(2)

Từ (1) và (2) suy ra: . Chú ý: .

Trong tam giác ABS có thẳng hàng.

b. Có CI là trung trực của ED nên tam giác PDE cân tại P.

+

+ Giả sử là điểm thỏa mãn suy ra Q, I, E thẳng hàng.

+ Có .

, (3)

Nhưng , (4)

+ Từ (3) và (4) suy ra tứ giác nội tiếp

hay

+ Suy ra tức Suy ra điều phải chứng minh.

  1. (Đề thi đề xuất chọn HSG vùng duyên hải đồng bằng Bắc Bộ năm 2015 - trường THPT chuyên Vĩnh Phúc) Cho tam giác nhọn không cân, nội tiếp đường tròn . là điểm nằm trong tam giác sao cho . Đường tròn đường kính cắt các cạnh lần lượt tại và cắt đường tròn tại điểm khác . Chứng minh rằng đồng quy.

Hướng dẫn giải:

Gọi AD là đường kính của (O), dễ thấy G,P,D thẳng hàng và . Giả sử PE,PF cắt DB,DC tại K,L; EF cắt BC tại T.

Theo định lý Desargues để chứng minh BE, CF, GP (hay PD) đồng quy ta chỉ cần chứng minh T,K,L thẳng hàng.

Áp dụng định lý Menelaus ta được: (1)

Dễ thấy tứ giác EFBC nội tiếp nên (2)

Cũng từ EFBC nội tiếp suy ra

Tứ giác PKDL là hình bình hành suy ra .

Suy ra (3).

Ta có (4)

Thay (2), (3), (4) vào (1) ta được . Từ đó áp dụng định lý menelaus cho tam giác DBC ta suy ra T,K,L thẳng hàng. Bài toán được chứng minh.

  1. (Đề thi đề xuất trường THPT chuyên tỉnh Hà Giang, trại hè Hùng Vương lần thứ XII) Cho tam giác cân tại . Một đường tròn tiếp xúc với các cạnh và cắt cạnh lần lượt tại và . Đoạn cắt đường tròn tại . Gọi và lần lượt là điểm đối xứng của qua và . Gọi là tâm đường tròn ngoại tiếp tam giác . Chứng minh rằng các điểm và tâm đường tròn thẳng hàng.

Hướng dẫn giải:

Gọi I là tâm của ; D, E theo thứ tự là tiếp điểm của và AB, AC; (O) là đường tròn ngoại tiếp tam giác MPQ.

Dễ thấy tứ giác MDKE điều hòa. Do đó

Dễ thấy DE // PK, mà nên .

Vậy. Từ đó hay M, D, P thẳng hàng.

Chứng minh tương tự M, E, Q thẳng hàng.

Kết hợp với suy ra

Do đó qua phép vị tự tâm M tỉ số k các điểm M, D, E theo thứ tự biến thành các điểm M, P, Q.

Vậy qua phép vị tự tâm M đường tròn biến thành đường tròn (O).

Do đó M, I, O thẳng hàng.

  1. (Đề thi đề xuất trường THPT chuyên Lê Quý Đôn, tỉnh Lai Châu, trại hè Hùng Vương lần thứ XII) Cho nhọn, các đường cao cắt nhau tại . Cho là một điểm tùy ý trên cạnh ( khác ). Kẻ đường kính của đường tròn ngoại tiếp tam giác và đường kính của đường tròn ngoại tiếp tam giác . Chứng minh rằng thẳng hàng.

Hướng dẫn giải:

Gọi L là giao điểm thứ hai của hai đường tròn (BKF) và (CKE).

Ta có tứ giác BFEC nội tiếp. Do đó A thuộc trục đẳng phương của hai đường tròn (BFK) và (CEK). Suy ra A, L, K thẳng hàng.

Vì tứ giác BFHD nội tiếp nên . Do đó tứ giác DHLK nội tiếp. Suy ra .

Mà nên M, H, L thẳng hàng.

Tương tự N, H, L thẳng hàng. Từ đó suy ra M, H, N thẳng hàng.

LOẠI 1: Chứng minh tính chất: thẳng hàng, đồng quy, song song, vuông góc.

Câu [TRƯỜNG THPT CHUYÊN BẮC GIANG]

Cho tam giác ngoại tiếp đường tròn tâm Đường tròn tiếp xúc với các cạnh lần lượt tại Gọi là giao của và là giao của và, là giao của và. Gọi lần lượt là trung điểm của và. Chứng minh rằng vuông góc, với là tâm đường tròn ngoại tiếp tam giác

Hướng dẫn giải

Xét 2 đường tròn: và

Ta có . (1) Gọi là bán kính đường tròn. Vì lần lượt là tiếp điểm của đường tròn nội tiếp (C­1) với các cạnh nên đồng quy.

Suy ra .

Ta có .

Khi đó:

, (2) Từ (1) và (2) suy ra là trục đẳng phương của và

Câu ĐỀ THI HỌC SINH GIỎI ĐB&DHBB

Cho tam giác nhọn nội tiếp đường tròn và có. Giả sử là trực tâm tam giác, đường tròn ngoại tiếp tam giác cắt tại điểm thứ hai là (). Đường thẳng đi qua, vuông góc với cắt tại F và cắt đường tròn tại hai điểm

a) Chứng minh tứ giác là tứ giác nội tiếp.

b) Chứng minh thẳng hàng.

Hướng dẫn giải

Giả sử ta có hình vẽ như trên (các trường hợp khác tương tự).

Gọi K là giao điểm thứ hai của CH và đường tròn

Ta có D là trung điểm KH và D cũng là trung điểm IJ, suy ra tứ giác IHJK là hình bình hành.

Suy ra (1).

Lại có tứ giác BHCE nội tiếp, suy ra . Mà .

Suy ra , như vậy tam giác cân tại C, nên D là trung điểm AE.

Suy ra tứ giác IAJE là hình bình hành và (2).

Lại có (3) (do tứ giác IAJK nội tiếp).

Từ (1), (2), (3) suy ra , như vậy tứ giác IHJE nội tiếp. Giả sử và là đường tròn ngoại tiếp các tứ giác IHJE và BHCE. Ta có HE chính là trục đẳng phương của và . Lại có (cùng là ).

Mà ;. Suy ra .

Suy ra F thuộc HE là trục đẳng phương của và . Vậy H, E, F thẳng hàng.

Câu TRƯỜNG THPT CHUYÊN NGUYỄN TẤT THÀNH

Cho tam giác có đường tròn nội tiếp tâm I, tiếp xúc với các cạnh lần lượt tại các điểm. Đường thẳng AI cắt đường tròn tại M, N sao cho M nằm giữa A và N. Đường thẳng DM và EF cắt nhau tại K, đường thẳng NK cắt đường tròn tâm tại điểm thứ hai là P khác N. Đường thẳng AI và EF cắt nhau tại Q.

a) Chứng minh rằng: Tứ giác là tứ giác nội tiếp

b) Chứng minh rằng: Các điểm thẳng hàng.

Câu HỘI CÁC TRƯỜNG CHUYÊN VÙNG DUYÊN HẢI VÀ ĐỒNG BẰNG BẮC BỘ

Cho hai đường tròn và cắt nhau tại . lần lượt là các đường kính của và . Gọi là trung điểm của ; là điểm thuộc đường phân giác của góc sao cho không vuông góc với và không thuộc hai đường tròn. Đường thẳng đi qua vuông góc với lần lượt cắt các đường tròn , tại các điểm khác . cắt đường tròn tại điểm thứ hai , cắt đường tròn tại điểm thứ hai .

1. Gọi là giao điểm của với . Chứng minh rằng là tiếp tuyến của đường tròn ngoại tiếp tam giác .

2. Chứng minh rằng ba đường thẳng và đồng quy.

Hướng dẫn giải

1. Không mất tính tổng quát giả sử là điểm thuộc đường phân giác trong của góc .

Ta có tứ giác là hình bình hành nên suy ra

Lại có

Do đó thẳng hàng. Chứng minh tương tự ta có thẳng hàngMặt khác

Do đó là tiếp tuyến của đường tròn 2. Ta có nên 4 điểm cùng thuộc đường tròn đường kính .

Mà nên suy ra là tiếp tuyến của đường tròn đường kính .

Do đó (1) Mặt khác

(2)

Từ (1) và (2) suy ra

Vậy 4 điểm cùng thuộc một đường tròn. Gọi là giao điểm của và

Vì 4 điểm cùng thuộc một đường tròn nên ta có

(3) Ta có (4) Gọi là giao điểm của với

Chứng minh tương tự câu 1) ta có là tiếp tuyến của đường tròn

Mặt khác tứ giác là hình thang vuông tại và là trung điểm của nên suy ra . Do đó (5) Từ (3), (4), (5) suy ra cùng thuộc trục đẳng phương của hai đường tròn nên thẳng hàng. Vậy 3 đường thẳng đồng quy tại

*) Chú ý: Nếu HS không sử dụng góc định hướng thì phải xét các trường hợp vị trí của điểm ( nằm ngoài các đoạn và nằm trong các đoạn )

II. Bài toán vecto và quan hệ vuông góc

  1. (Sở GDĐT Nghệ An- thi chọn học sinh giỏi tỉnh 2000-2011 toán 10)

a. Cho tứ giác lồi và điểm thỏa mãn hệ thức . Có kết luận gì về điểm , hãy chứng minh điều đó.

b. Cho tam giác có các đường trung tuyến và . Chứng minh rằng đường cao nằm trên trục đẳng phương của hai đường tròn đường kính và .

  1. (Sở GDĐT Nghệ An- thi chọn học sinh giỏi tỉnh 2004-2005)

a. Cho tứ giác , với , , . Chứng minh rằng:

b. Cho tam giác và , , lần lượt là trung điểm các cạnh , , . Dựng ra phía ngoài tam giác các đoạn thẳng , sao cho , và , . Từ dựng đường thẳng song song và cùng hướng với sao cho .

Chứng minh .

  1. (THPT Chuyên Bắc Giang – Tỉnh Bắc Giang – Thi Toán Khối 11)

Cho tam giác . Gọi là điểm thỏa mãn và giả sử . Gọi là hình chiếu vuông góc của trên , là trung điểm của đoạn . Chứng minh .

Lời giải

Gọi D là điểm đối xứng với B qua CK. Khi đó . Do nên

=nên

và tứ giác CHDA là tứ giác nội tiếp⇒

Do AH và BD cùng vuông góc với CK nên AH // DB⇒

Từ (1) và (2) suy ra tứ giác CEFD là tứ giác nội tiếp (DH cắt BC tại E, CK cắt BC tại F) suy ra H là trực tâm tam giác BCD ⇒ BH ⊥ CD⇒ BH//AD

Vậy tứ giác AHBD là hình bình hành, do M là trung điểm của AB nên H, M, D thẳng hang. Vậy MH ⊥ BC.

  1. (Sở GDĐT Nghệ An- thi chọn học sinh giỏi tỉnh 2000-2001 lớp 10)

a. Cho tam giác có ba điểm , , là trung điểm các cạnh , , . Tính giá trị biểu thức .

b. Cho tam giác có , ; . và là phân giác trong cắt nhau tại . Tính .

  1. ( Sở GDĐT Nghệ An- thi chọn học sinh giỏi tỉnh 2005-2006 lớp 12)

Cho tam giác cân đỉnh . Gọi là trung điểm của , là trọng tâm tam giác và là tâm đường tròn ngoại tiếp tam giác . Chứng minh .

IV. Bài toán chứng minh ba điểm thẳng hàng và tính chất đồng quy

  1. (THPT Chuyên Cao Bằng – Thi Olympic 2014 – Toán 11)

Cho hai đường tròn và ngoài nhau. Kẻ tiếp tuyến chung ngoài , tiếp tuyến chung trong của hai đường tròn (, . Chứng minh rằng , , đồng quy.

  1. (THPT Chu Văn An – Hà Nội – Đề xuất đề thi học sinh giỏi Toán 11- 2015)

Cho tam giác ABC nhọn, nội tiếp đường tròn . Kẻ đường kính . thuộc thỏa mãn . cắt tại khác . Chứng minh , , thẳng hàng với là trực tâm tam giác .

Lời giải

DP cắt AB tại E thì M là trung điểm DE (vì OM là đường trung bình)

BHCD là hình bình hành nên DH cắt DC tại I là trung điểm mỗi đường

Suy ra MI là đường trung bình của ∆DHE → MI // EH

→ EH // BC

Kéo dài CH cắt (O) tại Q. Ta sẽ c/m Q ≡ P, bằng cách c/m Q, E, D thẳng hàng.

Vì BD // CQ nên BDCQ là hình thang cân (hình thang nội tiếp).

Ta có: vì ∆QBH cân tại B

vì hình thang BDCQ cân

Nên

Mà Q, H, C thẳng hàng, nên E, Q, D thẳng hàng, hay QP (đpcm

  1. (THPT Chuyên tỉnh Lào Cai – Trại hè Hùng Vương lần X)

Cho đường tròn tâm và hai đường kính , . Tiếp tuyến với đường tròn tại cắt tại , cắt đường tròn lần nữa tại . Gọi là giao điểm của và . Chứng minh ba điểm , , thẳng hàng.

  1. ( THPT Chuyên Hưng Yên- Thi Môn Toán Khối 11 -2015)

Cho tam giác nội tiếp đường tròn . Đường phân giác trong của góc cắt tại khác . Gọi là điểm đối xứng với qua . cắt tại khác . là một điểm thay đổi trên cạnh . Đường thẳng cắt tại khác . Từ kẻ đường thẳng song song với cắt tại . Đường tròn ngoại tiếp cắt tại và cắt tại . Chứng minh , , thẳng hàng và đương thẳng luôn đi qua điểm cố định khi thay đổi.

Lời giải

+) là điểm chính giữa của cung của và E đối xứng qua nên

(vì ).

(g.c.g) là trung trực

(do ).

Mà (do )

và cùng phía đối với nên thẳng hàng.

+) Đường thẳng cắt tại khác

(vì ).

(vì ).

là đường kính của .

Mà là điểm chính giữa cung chứa thì không nên cố định. Vậy đường thẳng luôn qua điểm cố định khi thay đổi.

  1. (THPT Chuyên tỉnh Thái Nguyên – Trại hè Hùng Vương lần X)

Cho tam giác nhọn có là đường cao. , là chân đường vuông góc hạ từ xuống , . Gọi là giao của và , là giao điểm của đường tròn ngoại tiếp tam giác và , là tâm đường tròn ngoại tiếp tam giác . Chứng minh , , thẳng hàng.

Lời giải

Dễ dàng chứng minh được PQCB là tứ giác nội tiếp.

Ta có MB.MC = MP.MQ. Do hai tam giác MHPMQH đồng dạng nên .

Vậy có suy ra AKPH là tứ giác nội tiếp. Vậy .

Ta có năm điểm A, K, P, H, Q cùng thuộc đường tròn tâm J là trung điểm của AH. Bây giờ ta lại có I là tâm đường tròn ngoại tiếp tứ giác PQCB nên . Gọi O là tâm đường tròn ngoại tiếp tam giác ABC ta dễ dàng chứng minh được . Từ đó OA // IJ.

Lại có . Từ đó AJIO là hình bình hành. Dễ dàng suy ra JHIO cũng là hình bình hành. Mà JO vuông góc với AK (do AK là trục đẳng phương của hai đường tròn (J) và (O)). Vậy HI cũng vuông góc với AK. Lại có KH vuông góc với AK nên K, H, I thẳng hàng.

  1. (THPT Chuyên tỉnh Tuyên Quang – Trại hè Hùng Vương lần X)

Cho hai đường tròn và tiếp xúc với nhau tại , nằm trong , là một dây cung của tiếp xúc với tại . Gọi là tâm đường tròn nội tiếp tam giác . Chứng minh rằng

a. Ba điểm , , thẳng hàng.

b. Khi dây thay đổi thì điểm thuộc một đường tròn cố định.

Lời giải

a) Gọi M’ là giao điểm thứ hai của MA với (O) và B’ là giao điểm thứ hai của BA với (O’) (khác A).

Đặt . Ta thấy , suy ra O’M // OM’. Vì O’M ⊥ BC nên OM’ ⊥ BC, do đó M’ là điểm chính giữa cung . Vậy AM là phân giác của góc , hay I thuộc đường thẳng AM.

b) Theo tính chất của phân giác thì .

Mặt khác, theo tính chất của phương tích thì

.

Vì nên

.

Do đó

,

Vậy . Do nên cố định.

  1. THPT Chuyên Nguyễn Tất Thành – Tỉnh Yên Bái – Thi Toán Khối 11)

Cho tam giác không cân tại . Gọi và theo thứ tự là đường tròn ngoại tiếp, đường tròn nội tiếp tam giác . tiếp xúc với , tại , . Các điểm , thuộc sao cho song song và song song . Gọi , lần lượt là giao điểm của , với . Chứng minh rằng

a. , , đồng quy tại một điểm, gọi đó là điểm .

b. Đường tròn ngoại tiếp các tam giác , cùng tiếp xúc với và cùng đi qua một điểm thuộc .

Lời giải

a) Gọi S là giao điểm của BCEF. Gọi D là tiếp điểm của (I) với BC. Ta có DMFP là tứ giác điều hòa

EM//DS.

Do đó EP đi qua trung điểm của DS.

Tương tự FQ đi qua trung điểm của DS.

Vậy BC, EP, FQ đồng quy tại trung điểm của DS. Kí hiệu là K.

b) Kí hiệu (XYZ) là đường tròn ngoại tiếp tam giác XYZ.

Gọi d là tiếp tuyến vơi (I) tại P. Ta có:

.

Suy ra d tiếp xúc với (BPK) tại P. Vậy (PBK) tiếp xúc với (I).

Tương tự (CQK) tiếp xúc với (I).

Kí hiệu EE, FF theo thứ tự chỉ tiếp tuyến với (I) tại E, F. Gọi L là giao điểm khác K của (BPK) và (CQK). Ta có:

Suy ra L thuộc (ABC). Điều đó có nghĩa là đường tròn ngoại tiếp các tam giác BPKCQK cùng đi qua một điểm thuộc (O).

  1. (Sở GDĐT Nghệ An – Chọn đội tuyển dự thi HSG quốc gia lớp 12- 2006-2007)

Cho tam giác . Gọi hai điểm , lần lượt là trung điểm các cạnh , và là hình chiếu của lên . Chứng minh đường tròn ngoại tiếp các tam giác , , đồng quy tại một điểm đồng thời đường thẳng đi qua điểm đó và điểm H đi qua trung điểm đoạn thẳng .

Loại 2: Chứng minh tính chất: tam giác, tứ giác, đường tròn.

  1. [SỞ THỪA THIÊN HUẾ ( Vòng 2)- năm học 2001-2002]

Trong mặt phẳng, cho tứ giác (lồi) có: tổng khoảng cách từ mỗi đỉnh đến các cạnh là một số không đổi đối với tất cả các đỉnh. Chứng minh rằng tứ giác đó là hình bình hành.

Lời giải

Gọi là vectơ pháp tuyến đơn vị của đường thẳng a, có gốc trên a. M và N là hai điểm ở về

a

K

N

M

H

một nữa mặt phẳng có bờ a chứa vectơ .

Khi đó ta có: (1) (2)

Từ giả thiết ta được: tM = d(M,a) và tN = d(N,a) và từ (1) và (2) suy ra: (3).

Gọi , , , là các vectơ pháp tuyến đơn vị có gốc trên

các cạnh AB,BC,CD,DA và ở miền trong tứ giác ABCD.

Gọi k là tổng khoảng cách từ một đỉnh đến các đường thẳng chứa cạnh của tứ giác.

Khi đó ta có:

(+++)

= [d(B,AB) – d(A,AB)] + [d(B,BC) – d(A,BC)] + [d(B,CD) – d(A,CD)] + [d(B,DA) – d(A,DA)]

Do đó: (+++) = k – k = 0 (4). Tương tự ta có: (+++) = 0 (5).

Vì A,B,C không thẳng hàng nên từ (4) và (5) ta suy ra: +++ = (6).

Từ (6) suy ra: ++ = ++ nên: (,) = (,) (7).

Từ (6) suy ra: ++ = ++ nên: (,) = (,) (8).

Do: (,) + (,) + (,) + (,) = 3600 nên:

(,)+(,) = (,)+(,) = 1800, suy ra:(,) = 1800, tương tự: (,) = 1800. (9)

Từ (9) suy ra các cạnh đối của tứ giác song song nhau: AB // CD, BC // AD. Vậy ABCD là hình bình hành.

Cách khác: Sau khi chứng minh được (6). Gọi 0 là một điểm tùy ý.

Đặt: suy ra Ni thuộc đường tròn tâm O, bán kính 1. Do (6) suy ra O là trọng tâm của tứ giác N1N2N3N4 suy ra O là trung điểm của đoạn nối hai trung điểm của hai cạnh N1N2, N3N4 và từ đó suy ra: N1N2 // N3N4, suy ra N1N2N3N4 là hình chữ nhật, suy ra:

nên AB // CD,BC // AD. Vậy ABCD là hình bình hành.

  1. [SỞ THỪA THIÊN HUẾ ( Bảng B-Vòng 2)- năm học 2000-2001]

Cho đường thẳng cố định a và một điểm A cố định trên a. Gọi (C) là đường tròn lưu động ở trong một nữa mặt phẳng (α) có bờ a. (C) có bán kính không đổi R và luôn tiếp xúc với a, gọi M là tiếp điểm. Gọi I là tâm của đường tròn (C).

Chứng minh rằng trong mặt phẳng chứa đường tròn (C), có một parabol (P) cố định sao cho trục đẳng phương của (C) và đường tròn đường kính AI luôn luôn tiếp xúc (P) khi M thay đổi trên a.

Trong mặt phẳng chọn hệ trục tọa độ Đề-các vuông góc Oxy, với Ox trùng với a, nữa mặt phẳng α là nữa mặt phẳng y > 0, O trùng A. Đặt M(m;0) có tâm I(m;R).

Phương trình của (C) là:

M

I

x

y

A

(C): (x - m)2 + (y - R)2 = R2 hay

(C): x2 + y2 – 2mx – 2Ry + m2 = 0.

Phương trình đường tròn đường kính AI là:

(C’): (x – m/2)2 + (y – R/2)2 = hay

(C’): x2 + y2 – mx – Ry = 0.

Phương trình trục đẳng phương của hai đường tròn

(C) và (C’) là:

(d): mx + Ry – m2 = 0 ⇔ (d): y = f(x) = -.

Xét hàm số y = g(x) = .

Hệ .

Vậy Parabol y = f(x) = luôn tiếp xúc với trục đẳng phương (d).

Loại 2: Chứng minh các tính chất:tam giác, tứ giác đường tròn.

  1. [ĐỀ THI HỌC SINH GIỎI TRẠI HÈ HÙNG VƯƠNG LẦN THỨ IX TRƯỜNG THPT CHUYÊN HOÀNG VĂN THỤ-2013]

Cho cân tại . Gọi là trung điểm . Đường tròn ngoại tiếp tam giác giao với phân giác góc tại nằm trong . Đường tròn ngoại tiếp giao với tại ( khác ), giao với tại . giao với tại . Chứng minh rằng là tâm đường tròn nội tiếp tam giác .

  1. [ ĐỀ DUYÊN HẢI LỚP 11 MÔN TOÁN – Trường THPT Nguyễn Trãi, tỉnh Hải Dương]

Cho tam giác nội tiếp đường tròn tâm và có trực tâm . là một điểm bất kì trên . Gọi là các điểm đối xứng với lần lượt qua trung điểm các cạnh . Chứng minh rằng bốn điểm nằm trên một đường tròn.

Hướng dẫn giải

- Gọi lần lượt là trung điểm của .

Xét phép vị tự

Suy ra phép vị tự này biến đường tròn ngoại tiếp tam giác hay đường tròn

Ơle của tam giác thành đường tròn ngoại tiếp tam giác .

Từ đó để chứng minh thuộc đường tròn ta chứng minh thuộc đường tròn Ơle của tam giác .

- Xét phép vị tự , suy ra phép vị tự này biến đường tròn Ơle của tam giác thành đường tròn , mà nằm trên đường tròn Ơle của tam giác (đpcm).

  1. [TRƯỜNG THPT CHUYÊN LÀO CAI TỔ TOÁN – TIN HỌC ĐỀ THI ĐỀ XUẤT KỲ THI HSG VÙNG DUYÊN HẢI VÀ ĐỒNG BẰNG BẮC BỘ LẦN THỨ VII. MÔN TOÁN: KHỐI 11.Năm học: 2013-2014 ]

Cho tam giác nhọn không cân. Gọi lần lượt là trực tâm, tâm đường tròn ngoại tiếp tam giác; lần lượt là chân đường cao hạ từ các đỉnh của tam giác. Các đường thẳng và cắt nhau tại , các đường thẳng và cắt nhau tại . Gọi là trung điểm cạnh . Chứng minh rằng ba điểm thẳng hàng khi và chỉ khi bốn điểm cùng nằm trên một đường tròn.

Áp dụng đinh lý Mê-nê-la-uýt cho tam giác và ba điểm ta có:

thẳng hàng khi và chỉ khi

Ta lại có (cùng cạnh đáy ), (cùng cạnh đáy ) và .(Bởi vì ,

ở đó là bán kính đường tròn ngoại tiếp tam giác và . Tương tự ).

Từ các kết quả trên ta có khi và chỉ khi hoặc đi qua trung điểm .

Bằng cách vẽ tiếp tuyến của đường tròn ngoại tiếp tam giác tại , dễ dàng suy ra , suy ra vuông góc với .

Gọi lần lượt là trung điểm của . Dễ thấy tứ giác nội tiếp, suy ra vuông góc với . Suy ra .

Nếu đi qua trung điểm suy ra là trung điểm , suy ra là hình bình hành, suy ra và . Điều này trái với giả thiết cắt và cắt .

Vậy xảy ra khi và chỉ khi khi và chỉ khi vuông góc với khi và chỉ khi cùng nằm trên một đường tròn (vì ta luôn có tứ giác nội tiếp đường tròn đường kính ).

  1. [TRẠI HÈ HÙNG VƯƠNG LẦN THỨ XI TRƯỜNG THPT CHUYÊN LÊ QUÝ ĐÔN TỈNH ĐIỆN BIÊN ĐỀ THI MÔN TOÁN LỚP 11 ]

Cho tam giác nhọn , các đường cao . Một đường tròn đi qua hai điểm và tiếp xúc với cạnh tại điểm . Đường thẳng cắt đường tròn ngoại tiếp tam giác tại điểm thứ hai . Hai đường thẳng và cắt nhau tại . Chứng minh rằng đường thẳng tiếp xúc với đường tròn ngoại tiếp tam giác .

Hướng dẫn giải

Ta có (cùng bằng một nửa số đo cung );

mà nên suy ra

suy ra và đồng dạng (g.g)

(1)

Nhận xét:

Trong tam giác nhọn tùy ý ta có: .

Thậy vậy

Kết hợp nhận xét trên với suy ra .

Lại có (cùng bằng phương tích của điểm đối với ), nên suy ra:

suy ra đường thẳng tiếp xúc với đường tròn ngoại tiếp tại điểm .

Vậy đường thẳng tiếp xúc với đường tròn ngoại tiếp .

  1. [Ngân hàng đề Hùng Vương-Trường CHUYÊN BẮC GIANG – năm-Tỉnh BẮC GIANG]

Cho tam giác nhọn không cân tại, là trung điểm cạnh, và tương ứng là chân đường cao hạ từ, của tam giác. là giao điểm của hai tiếp tuyến tại , của đường tròn ngoại tiếp tam giác, là giao điểm của và, là giao điểm của và

a) Chứng minh rằng là tâm đường tròn nội tiếp tam giác .

b) Đường tròn ngoại tiếp tam giác và cắt nhau tại điểm thứ hai . Chứng minh rằng trực tâm của tam giác nằm trên.

c) Chứng minh rằng D nằm trên đường tròn ngoại tiếp tam giác XYZ.

Hướng dẫn giải

a) là phân giác góc.

Do và , từ đó và nằm trên trung trực của, do đó là tâm đường tròn nội tiếp tam giác

b)Giả sử, khi đó nằm trên cung nhỏ. Gọi là tâm đường tròn ngoại tiếp tam giác và là trung điểm của. Ta có được và vuông góc.

Từ và vuông góc, ta được và song song. là hình bình hành nên song song với, do đó , , thẳng hàng.

c) Chứng minh được góc và là đường trung trực của .

Chứng minh được góc nên là tứ giác nội tiếp.

Do đó góc , do đó là tứ giác nội tiếp.

  1. [THI HSG LỚP 11 THPT NĂM HỌC 2010-2011-THPT CHUYÊN VĨNH PHÚC ]

Cho tam giác ABC. Phân giác trong của các góc A, B, C cắt đường tròn ngoại tiếp tam giác ABC lần lượt tại các điểm. Đường thẳng cắt đường thẳng tại điểm ; đường thẳng cắt đường thẳng tại điểm ; đường thẳng cắt đường thẳng tại điểm . Gọi O là tâm đường tròn ngoại tiếp tam giác . Đường thẳng cắt đường thẳng tại điểm M. Biết rằng và . Tính các góc của tam giác ABC.

Hướng dẫn giải

* Dễ thấy , do đó O là trung điểm của .

*

* Do BM=MN;

Do đó , mà

Vậy

Cùng với ta được

Trường hợp ta cũng chỉ ra được

(tính chất phép vị tự).

(góc tạo bởi tiếp tuyến và dây cung chắn hai cung bằng nhau) và DE = QF.

Lại có CE = CF theo tính chất của hai tiếp tuyến kẻ từ một điểm.

Suy ra , dẫn đến . Từ đó ta có điều phải chứng minh.

  1. [§Ò thi hsg 11 tØnh nghÖ an - N¨m häc 2002-2003]

Trong tam gi¸c ABC, D lµ trung ®iÓm cña c¹nh BC. Gi¶ sö gãc . H·y tÝnh tanC.

Hướng dẫn giải

§Æt BD =CD = a vµ AC =b. ¸p dông ®Þnh lý sin trong ΔACD vµ ΔABC ta cã:

  1. [TRƯỜNG THPT NGUYỄN HỮU THẬN QUẢNG TRỊ Năm học: 2011 – 2012]

Trong mặt phẳng tọa độ (Oxy) cho đường tròn () có tâm , bán kính và đường tròn () có tâm và bán kính . Xác định phép vị tự tâm I, tỉ số k biến () thành () biết I nằm giữa hai điểm .

Hướng dẫn giải

Theo giải thiết suy ra . Mặt khác nằm khác phía đối với I nên suy ra .

Gọi , ta có Vậy I(1;2)

  1. [KỲ THI OLYMPIC 30-4 Lần 16 (3-4-2010) Toán Khối 11]

Hãy tìm bên trong một tứ giác lồi một điểm sao cho các đoạn thẳng nối điểm đó và trung điểm các cạnh đối diện chia tứ giác thành 4 phần có diện tích bằng nhau bằng nhau.

Hướng dẫn giải

  1. [Ngân hàng đề Hùng Vương-Trường CHUYÊN BẮC GIANG – năm-Tỉnh BẮC GIANG]

Cho tam giác nhọn không cân tại, là trung điểm cạnh, và tương ứng là chân đường cao hạ từ, của tam giác. là giao điểm của hai tiếp tuyến tại , của đường tròn ngoại tiếp tam giác, là giao điểm của và, là giao điểm của và

a) Chứng minh rằng là tâm đường tròn nội tiếp tam giác .

b) Đường tròn ngoại tiếp tam giác và cắt nhau tại điểm thứ hai . Chứng minh rằng trực tâm của tam giác nằm trên.

c) Chứng minh rằng D nằm trên đường tròn ngoại tiếp tam giác XYZ.

Hướng dẫn giải

a) là phân giác góc.

Do và , từ đó và nằm trên trung trực của, do đó là tâm đường tròn nội tiếp tam giác

b)Giả sử, khi đó nằm trên cung nhỏ. Gọi là tâm đường tròn ngoại tiếp tam giác và là trung điểm của. Ta có được và vuông góc.

Từ và vuông góc, ta được và song song. là hình bình hành nên song song với, do đó , , thẳng hàng.

c) Chứng minh được góc và là đường trung trực của .

Chứng minh được góc nên là tứ giác nội tiếp.

Do đó góc , do đó là tứ giác nội tiếp.

  1. [Ngân hàng đề Tr­êng T.H.P.T Chuyªn Th¸i B×nh- ĐỀ ĐỀ NGHỊ THI CHỌN HSG

VÙNG DUYÊN HẢI BẮC BỘ LỚP 11

N¨m häc 2013-2014]

Cho tam giác ABC vuông tại A. Hình chữ nhật MNPQ thay đổi sao cho M thuộc AB, N thuộc AC và P, Q thuộc BC. K = BN ∩ MQ; L = CM ∩ NP;X =MP ∩ NQ;Y =KP ∩ LQ Chứng minh rằng.

1)

2) XY luôn đi qua một điểm cố định.

Hướng dẫn giải

1) Lấy U, V theo thứ tự thuộc AK, AL sao cho (h.1).

(h.2.1)

Do đó các tam giác ABU, ACV đồng dạng.

Vậy

2)Đặt Z = ML ∩ NK (h.2.2).

Theo định lí Pappus, X, Y, Z thẳng hàng (1).

Gọi H là hình chiếu của A trên BC; O, F, E theo thứ tự là trung điểm của BC, MN,AH.

Dễ thấy A, Z, O, F thẳng hàng; E, X, O thẳng hàng; FX // AH.

Vậy

Do đó X, H, Z thẳng hàng (2).

Từ (1) và (2) suy ra XY đi qua H (đpcm).

(h.2.2)

  1. Cho đường tròn (O, R) và một điểm S ở trong đường tròn. Xét tất cả các góc vuông đỉnh S: gọi giao điểm của hai cạnh góc vuông với đường tròn là A, B. Tìm tập hợp trung điểm M của AB.

Hướng dẫn giải

Gọi I là trung điểm SO, đặt (không đổi)

Ta có

Mặt khác mà

nên

không đổi.

Vì I cố định nên M thuộc đường tròn tâm I bán kính

Đảo lại, trên đường tròn (I) lấy M’ tuỳ ý . Lấy M’ làm tâm quay một cung tròn bán kính M’S cắt (O) tại A’.

Ta có M’A’ = M’S. Kéo dài M’A’ cắt (O) tại B’.

Xét tam giác M’SO có:

Hay tam giác OM’A’ vuông tại M’ suy ra M’ là trung điểm A’B’

nghĩa là M’A’=M’B’=M’S hay tam giác SA’B’ vuông tại S.

  1. [Ngân hàng đề Trường THPT chuyên Nguyễn Trãi, tỉnh Hải Dương - ĐỀ ĐỀ NGHỊ THI CHỌN HSG VÙNG DUYÊN HẢI BẮC BỘ LỚP 11 N¨m häc …]

()

Cho tam giác ABC nội tiếp đường tròn tâm O và có trực tâm H. P là một điểm bất kì trên (O). Gọi Q, R, S là các điểm đối xứng với P lần lượt qua trung điểm các cạnh BC, CA, AB. Chứng minh rằng bốn điểm H, Q, R, S nằm trên một đường tròn.

Hướng dẫn giải

- Gọi M, N, K, I lần lượt là trung điểm của BC, CA, AB, PH. Xét phép vị tự

Suy ra phép vị tự này biến đường tròn ngoại tiếp tam giác MNP hay đường tròn

Ơle của tam giác ABC thành đường tròn ngoại tiếp tam giác QRS.

Từ đó để chứng minh H thuộc đường tròn (QRS) ta chứng minh I thuộc đường tròn Ơle của tam giác ABC.

- Xét phép vị tự , suy ra phép vị tự này biến đường tròn Ơle của tam giác ABC thành đường tròn (O), mà I nằm trên đường tròn Ơle của tam giác ABC (đpcm).

  1. [TRƯỜNG THPT CHUYÊN NGUYỄN BỈNH KHIÊM QUẢNG NAM

KỲ THI CHỌN HỌC SINH GIỎI KHU VỰC DUYÊN HẢI & ĐỒNG BẰNG BẮC BỘ NĂM 2013 ĐỀ THI ĐỀ NGHỊ MÔN: TOÁN, LỚP 11]

Cho đường tròn (O;R) và một điểm I cố định ở trong đường tròn (IO), đường thẳng qua I vuông góc với OI cắt đường tròn tại C và D; A là một điểm nằm trên đường tròn, tia đối xứng với tia IA qua đường thẳng CD cắt đường tròn tại B. Gọi M là trung điểm của AB.

a) Chứng minh đường thẳng AB đi qua một điểm cố định L khi A thay đổi trên đường tròn (O;R).

b) Gọi N, P là giao điểm của đường thẳng OM với đường tròn (O). Đường thẳng CN và DP cắt nhau ở Q. Chứng minh rằng các điểm Q, N là những tâm của đường tròn nội tiếp và bàng tiếp của tam giác CMD.

Hướng dẫn giải

Gọi L là giao điểm của AB và OI; K là giao điểm của AB và CD.

Ta có IE ⊥OL và IE là phân giác của góc , suy ra: (ABKL)=-1

Suy ra: (M là trung điểm của AB, New-tơn)

=

=

Mà ta lại có:

Do đó:

Suy ra:

Suy ra:

Suy ra . Suy ra . Vậy L cố định.

Trước hết ta chứng minh MK là phân giác của góc CMD.

Gọi E là giao điểm của OM với CD

Ta có: ΔOIE~ΔOML

Suy ra:

Suy ra:

Suy ra:

Ta có:

Do đó ta suy ra:

Suy ra:

Theo hệ thức Newton, ta suy ra: (CDKE)=-1 (1)

Mà MK ⊥ME nên MK là phân giác trong của góc (2)

Theo chứng minh trên ta có:

Suy ra: (PNME)= -1

Suy ra: (NPME)= -1 (3)

Từ (1) và (3) ta suy ra: CN, PD, KM đồng quy tại Q.

Mà góc nên QMND là tứ giác nội tiếp

Suy ra:

Suy ra DP là phân giác trong của góc . (4)

Từ (2) và (4), ta có Q là tâm đường tròn nội tiếp tam giác CMD

Ta lại có DN⊥DP suy ra DN là phân giác ngoài của góc. Suy ra N là tâm đường tròn bàng tiếp của tam giác CMD.

Vậy Q, N lần lượt là tâm của đường tròn nội tiếp và bàng tiếp của tam giác CMD.

  1. [TRƯỜNG THPT CHUYÊN NGUYỄN BỈNH KHIÊM TỈNH QUẢNG NAM NĂM 2015 ]

Cho nhọn có . Hai đường phân giác trong và ngoài của lần lượt cắt đường thẳng tại và ; hai đường phân giác trong và ngoài của lần lượt cắt đường thẳng tại . Giả sử hai đường tròn đường kính và gặp nhau tại một điểm nằm bên trong . Chứng minh rằng .

Hướng dẫn giải

Gọi là trung điểm của đoạn thẳng .

Khi đó hai điểm và nằm trên đường tròn tâm bán kính .

Vì nên ,

Suy ra .

Từ đó .

Do đó

Như vậy suy ra (1)

Tương tự ta cũng có . (2)

Ngoài ra (3)

Từ (1), (2) và (3) ta đi đến .

Suy ra .

  1. [TRƯỜNG THPT CHUYÊN LÊ QUÝ ĐÔN TỈNH ĐIỆN BIÊN TRẠI HÈ HÙNG VƯƠNG LẦN THỨ XI LỚP 11]

Cho tam giác nhọn ABC, các đường cao BD, CE. Một đường tròn (O) đi qua hai điểm A và E tiếp xúc với cạnh BC tại điểm M. Đường thẳng ME cắt đường tròn ngoại tiếp tam giác AED tại điểm thứ hai K. Hai đường thẳng DK và BC cắt nhau tại N. Chứng minh rằng đường thẳng BC tiếp xúc với đường tròn ngoại tiếp tam giác AEN.

Hướng dẫn giải

Ta có (cùng bằng một nửa số đo cung EM);

mà nên suy ra

suy ra và đồng dạng (g.g)

(1)

Nhận xét: Trong tam giác nhọn ABC tùy ý ta có: .

Thậy vậy

Kết hợp nhận xét trên với (1) suy ra .

Lại có (cùng bằng phương tích của điểm B đối với (O)), nên suy ra:

suy ra đường thẳng BN tiếp xúc với đường tròn ngoại tiếp tam giác AEN tại điểm N. Vậy đường thẳng BC tiếp xúc với đường tròn ngoại tiếp tam giác ANE.

  1. [Tr­ưêng PT CÊp 2+ 3 T©n S¬n B¾c giang N¨m häc 2008 – 2009]

Cho tam gi¸c ABC néi tiÕp ®­êng trßn (C). §iÓm A’ lµ ¶nh cña A qua phÐp ®èi xøng trôc BC. T×m quü tÝch ®iÓm A’ khi A di ®éng trªn (C).

Hướng dẫn giải

  1. [Chuyên Lê Quý Đôn - Đà Nẵng KỲ THI HỌC SINH GIỎI CÁC TRƯỜNG THPT CHUYÊN KHU VỰC DUYÊN HẢI VÀ ĐỒNG BẰNG BẮC BỘ

LẦN THỨ IX, NĂM HỌC 2015 – 2016]

Cho đường tròn và dây AB. Các đường tròn và nằm về một phía đối với đường thẳng AB, tiếp xúc với nhau tại T đồng thời tiếp xúc với AB và tiếp xúc trong với đường tròn . Tiếp tuyến chung tại T của các đường tròn và cắt đường tròn tại (với C thuộc nửa mặt phẳng với bờ là đường thẳng AB có chứa hai đường tròn và ).

Chứng minh rằng T là tâm đường tròn nội tiếp tam giác ABC.

Hướng dẫn giải

+ Gọi E, F, M, N lần lượt là tiếp điểm , với đường tròn và AB như hình vẽ. Gọi K là giao điểm thứ hai của EF với .

Ta có các điểm E, , O thẳng hàng; các điểm M, , O thẳng hàng.

+ Hơn nữa .

Vậy K là điểm chính giữa cung AB.

Như vậy EF đi qua điểm chính giữa K của cung AB.

+ Chứng minh tương tự ta cũng có MN cũng đi qua K.

+ Từ đó nên tứ giác EFNM là tứ giác nội tiếp, do đó

.

Vậy điểm K nằm trên trục đẳng phương của và , suy ra ba điểm C, T, K thẳng hàng.

Từ đó điểm T nằm trên phân giác của

+ Ta có các cặp tam giác đồng dạng và ; và .

Từ đó , suy ra .

Ta lại có , suy ra .

Vì vậy các tam giác KATKBT cùng cân tại K.

Do đó .

Suy ra AT là phân giác của (2)

+ Từ (1) và (2) suy ra T là tâm đường tròn nội tiếp tam giác ABC (đpcm).

  1. [CHUYÊN CAO BẰNG TRẠI HÈ HÙNG VƯƠNG NĂM 2013]

Cho hai đường tròn (O, R) và (O’, R’) với cắt nhau tại hai điểm phân biệt A và B. Một đường thẳng d tiếp xúc với đường tròn (O) và (O’) lần lượt tại P và P’. Gọi Q và Q’ lần lượt là chân đường vuông góc hạ từ P và P’ xuống OO’. Các đường thẳng AQ và AQ’ cắt các đường tròn (O) và (O’) tạiM và M’.Chứng minh rằng MM’ qua B.

Hướng dẫn giải

Gọi S là giao điểm của d và OO’, khi đó S là tâm vị tự ngoài của hai đường tròn (O) và (O’). Đặt , khi đó ta có.

Gọi I, J là giao điểm của AB với PP’ và OO’. Khi đó ta có

Mà PQ // IJ // P’Q’ nên JQ = JQ’

Suy ra AB là trung trực của QQ’.

Mà OO’ là trung trực của AB. Vậy tứ giác AQBQ’ là hình thoi

Do đó Q’B //AQ hay Q’M’ // QM.

Giả sử V(S, k) biến M thành B’ khi đó QM // Q’B’

Mà M thuộc (O) suy ra B’ thuộc (O’) do đó B’ trùng với B.

Vậy V(S, k) biến M thành B.

Tương tự ta có V(S, k) biến M’ thành B.

Suy ra M, B, M’ thẳng hàng hay MM’ qua B

  1. [Trường THPT Chuyên Hạ Long Đề thi Trại Hè Hùng Vương năm 2013]

Cho tam giác ABC cân tại A. Gọi D là trung điểm AC.Đường tròn ngoại tiếp tam giác BCD giao với phân giác góc tại E nằm trong tam giác ABC. Đường tròn ngoại tiếp tam giác ABE giao với BD tại F ( khác B), AF giao với BE tại I. CI giao với BD tại K. Chứng minh rằng I là tâm đường tròn nội tiếp tam giác ABK.

Hướng dẫn giải

Gọi D’ là trung điểm của AB và M là trung điểm cạnh BC.

Ta có D’ nằm trên đường tròn ngoại tiếp 🛆BCD. Do tính đối xứng nên suy ra suy ra

suy ra I nằm trên phân giác góc hay BI là tia phân giác góc (1) 1.0 đ

Ta có:

=> suy ra 🛆AFD cân tại D. 1.0 đ

Do IA.IF = IE.IB nên I thuộc trục đẳng phương của đường tròn đường kính AC và đường tròn ngoại tiếp .

Từ đó CI đi qua giao điểm thứ hai J của hai đường tròn này. 1,0đ

Ta có nên

Suy ra hay . Từ đó .

Ta có (đpcm) 1.0đ

  1. [CHUYÊN HÙNG VƯƠNG ĐỀ THI TRẠI HÈ HÙNG VƯƠNG 2013]

Cho tam giác nhọn nội tiếp trong đường tròn tâm O. Các điểm lần lượt thuộc các cạnh sao cho Đường tròn ngoại tiếp tam giác cắt cạnh tại và Chứng minh rằng:

a) là một tứ giác nội tiếp.

là tiếp tuyến của đường tròn ngoại tiếp tam giác

Hướng dẫn giải

Ký hiệu

a) Từ giả thiết, ta có

Suy ra là một tứ giác nội tiếp.

b) Trước hết, ta chứng minh là một tứ giác nội tiếp.

Thật vậy,

Từ đó hay là một tứ giác nội tiếp.

Do là một tứ giác nội tiếp nên

Ta chứng minh

thật vậy

Từ (1) và (2) suy ra là tiếp tuyến của đường tròn ngoại tiếp tam giác

  1. [Trường THPT Chuyên Hoàng Văn Thụ - Tỉnh Hòa Bình-ĐỀ XUẤT ĐỀ THI TRẠI HÈ HÙNG VƯƠNG Năm học 2012-2013]

Cho hai đường tròn (O) và (O’) cắt nhau tại A và B. M là một điểm thuộc đường tròn (O). MA cắt lại đường tròn (O’) lần thứ hai tại N, MB cắt lại đường tròn (O’) lần thứ hai tại P. Tiếp tuyến tại A và B của (O) cắt nhau tại Q. Chứng minh rằng MQ đi qua trung điểm của NP.

Hướng dẫn giải

Qua Q kẻ EF // NP (EMN, FMP)

Yêu cầu đề bài trở thành chứng minh Q là trung điểm EF.

Ta có:

(Do ANPB là tứ giác nội tiếp)

Lại có

cân tại Q

Chứng minh tương tự cân tại Q

Q là trung điểm EF

MQ đi qua trung điểm NP

  1. [TRƯỜNG THPT CHUYÊN HƯNG YÊN TỈNH HƯNG YÊN NĂM 2016]

Cho hai đường tròn (O1), (O2) cắt nhau tại hai điểm A, B. Một đường thẳng qua B cắt hai đường tròn (O1), (O2) tại điểm thứ hai là C, D. Gọi M là trung điểm của CD; Đường thẳng AM cắt đường tròn (O2) tại điểm thứ hai là P; Đường thẳng (d) qua M và vuông góc với O1M cắt đường thẳng AC tại Q. Chứng minh đường thẳng PQ đi qua một điểm cố định.

Hướng dẫn giải

Bổ đề: Tứ giác ABCD nội tiếp (O); Gọi M là giao điểm của AB và CD. Đường thẳng d qua M và d ⊥ OM. Đường thẳng d cắt BD, AC, AD, BC lần lượt tại K, H, I, J. Chứng minh MH = MK; MI = MJ.

Gọi N là giao điểm của ADBC; G là giao điểm của AC và BD.

Dựng hai tiếp tuyến NP, NQ của (O) (P, Q là các tiếp điểm).

PQ cắt BC, AD tại ZT.

Ta có: (NZCB) = (NTDA) = - 1 nên ZT, CD, AB đồng quy tại M.

(NZCB) = (NTAD) = - 1 nên ZT, CA, BD đồng quy tại G

Do đó P,Q, Z, T, G, M thẳng hàng.

Theo định lý Brocard, O là trực tâm tam giác GMN nên NG // d (1)

Ta có, (NTDA) = - 1 nên G(NTDA) = - 1 (2)

Từ (1) và (2) suy ra M là trung điểm của HK.

Ta có, ABCDMN là tứ giác toàn phần nên (XGDB) = -1

N(XGDB) = -1 (3).

Từ (1) và (3) suy ra M là trung điểm của IJ.

Trở lại bài toán:

Gọi K = AM ∩ (O1); R = BK ∩ d

Ta có CK // PDM là trung điểm của CD nên CKDP là hình bình hành. Suy ra M là trung điểm của PK.

Theo bổ đề, ta có: M là trung điểm của QR nên KRPQ là hình bình hành. Do đó, RK // PQ.

Gọi E là giao điểm của PQ và (O2).

Ta có: ; nên

Suy ra hay BE là tiếp tuyến của (O1) nên E cố định.

Vậy PQ luôn đi qua điểm E cố định.

  1. [TRƯỜNG THPT CHUYÊN LÊ QUÝ ĐÔN TỈNH ĐIỆN BIÊN NĂM 2016]

Cho tam giác ABC nhọn, không cân nội tiếp đường tròn (O) có đường cao AH và tâm đường tròn nội tiếp là I. Gọi M là điểm chính giữa cung nhỏ BC của (O) và D là điểm đối xứng với A qua O. Đường thẳng MD cắt các đường thẳng BC, AH theo thứ tự tại PQ.

a. Chứng minh rằng tam giác IPQ vuông.

b. Đường thẳng DI cắt (O) tại điểm E khác D. Hai đường thẳng AEBC cắt nhau tại điểm F. Chứng minh rằng nếu thì I là trọng tâm của tam giác APF.

Hướng dẫn giải

Các hình vẽ sau cho trường hợp AB < AC.

a. (3 điểm)

Ta có: ∠OAC = 900 – ∠AOC

= 900 –∠ABC = ∠BHA

AI là phân giác ∠BAC nên

HAI = ∠OAI.

Suy ra ΔAQD cân tại AMQ = MD (1).

Gọi L là giao điểm AMBC.

Khi đó ∠LPD = 900 – ∠HQP

= 900 – ∠ADM = ∠LAD.

Do đó tứ giác ALDP nội tiếp được

MD.MP = ML.MA (2).

Ta có ΔMLC ~ ΔMCA (g – g) nên ⇒ (3).

Lại có ∠MIC = ∠MCI = nên ΔMIC cân tại MMC = MI (4).

Từ (1), (2), (3), (4) ta có: = ML.MA = MD.MP = MQ.MP.

Suy ra tam giác IPQ vuông tại I.

b. (2 điểm)

Từ câu a. ta suy ra HIPQ nội tiếp

⇒ ∠IHP = ∠IQP = ∠IDM = ∠EAM

do đó tứ giác AIHF nội tiếp

⇒ ∠AIF = ∠AHF = 900.

Gọi N là trung điểm của đoạn FA.

Khi đó ∠NIA = ∠NAI = ∠EDM

= ∠IQP = ∠MIP

nên N, I, P thẳng hàng.

Theo tính chất phân giác và giả thiết thì

.

Áp dụng định lý Menelauyt cho tam giác AFL với cát tuyến NIP ta có

⇔ ⇒ L là trung điểm của PF.

Từ đó suy ra I là trọng tâm của tam giác APF.

  1. [TRƯỜNG THPT CHUYÊN HOÀNG VĂN THỤ TỈNH HÒA BÌNH TRẠI HÈ HÙNG VƯƠNG LẦN THỨ XII]

Cho tam giác ABC, đường tròn tâm O bàng tiếp góc A tiếp xúc với cạnh BC, CA, AB lần lượt tại T, F, E. Hai đường thẳng BE, CF cắt nhau tại I

a) Chứng minh A, I, T thẳng hàng

b) Vẽ đường tròn tâm O khác đường tròn bàng tiếp góc A ở trên cắt đoạn AB,AC tại M, N; cắt đường thẳng BC tại với thuộc tia đối BC, thuộc tia đối CB. M cắt N tại K. Chứng minh rằng K nằm trên đường thẳng AI

Hướng dẫn giải

a) Cần chứng minh AT, BF,CE đồng quy. Áp dụng định lí Ceva với tam giác ABC

suy ra A,I,T thẳng hàng.

b)

+

+

Cần chứng minh AM=AN

Ta có AO vuông góc với MN suy ra tam giác AMN cân dẫn đến AM=AN

  1. [TRƯỜNG THPT CHUYÊNLƯƠNG VĂN TỤY KHU VỰC DUYÊN HẢI VÀ ĐỒNG BẰNG BẮC BỘ NĂM HỌC 2014 – 2015]

Cho tam giác ABC ( ) có H là trực tâm và M là trung điểm của BC. P là một điểm thuộc đường thẳng HM, đường tròn (K) đường kính AP cắt AC, AB lần lượt tại E, F khác A. Chứng minh rằng tiếp tuyến tại E, F của (K) cắt nhau trên trung trực BC.

Hướng dẫn giải

Gọi T là giao điểm của các tiếp tuyến với (K) tại E, F. Y, Z theo thứ tự là giao điểm của CH, BH và CA, AB. (N) là đường tròn đường kính AH, S là giao điểm thứ hai của (K) và (N). Có hai trường hợp xảy ra:

Trường hợp 1. P trùng với H. Dễ thấy . Do đó.

Trường hợp 2. P không trùng với H.

Dễ thấy các tam giác SEY, SFZ đồng dạng cùng hướng.

Do đó các tam giác SEF, SYZ đồng dạng cùng hướng (1).

Từ (1), chú ý rằng TE, TF tiếp xúc với (K) tại E,F và MY, MZ tiếp xúc với (N) tại Y, Z suy ra các tam giác TEF, MYZ đồng dạng cùng hướng (2).

Từ (1), (2) suy ra các tam giác STF, SMZ đồng dạng cùng hướng.

Do đó các tam giác STM, SFZ đồng dạng cùng hướng (3).

Dễ thấy: . Do đó: SH=SP.

Kết hợp với P thuộc HM suy ra SM=SH(4).

Từ (3), (4) suy ra:

Vậy .

Nói cách khác, tiếp tuyến tại E, F của (K) cắt nhau trên trung trực BC.

  1. [TRƯỜNG THPT CHUYÊN NGUYỄN TRÃI TỈNH HẢI DƯƠNG NĂM 2015]

Cho tứ giác ABCD nội tiếp đường tròn tâm O và có AC vuông góc với BD tại điểm H. Gọi I, J, K, L lần lượt là hình chiếu vuông góc của H trên các đường thẳng AB, BC, CD, DA, gọi M, N, P, Q tương ứng là trung điểm của các cạnh AB, BC, CD, DA.

a) Chứng minh rằng tám điểm I, J, K, L, M, N, P, Q cùng nằm trên một đường tròn có tâm là trung điểm của đoạn thẳng OH.

b) Chứng minh rằng giao điểm của IK và JL nằm trên đường thẳng OH.

Hướng dẫn giải

Trước hết ta chứng minh H, I, P thẳng hàng. Thật vậy, ( vì HP là trung tuyến của tam giác vuông HCD)

suy ra I, H, P thẳng hàng. Tương tự cũng có J, H, Q thẳng hàng, K, H, M

thẳng hàng, L, H, N thẳng hàng.

Rõ ràng I và K nằm trên đường tròn đường kính MP, J và L nằm trên đường

tròn đường kính NQ. (1)

Mặt khác, MNPQ là hình bình hành và MN song song AC, NP song song

BD, AC vuông góc BD nên MN vuông góc NP, vì vậy MNPQ là hình chữ

nhật. Do đó đường tròn đường kính MP cũng là đường tròn đường kính NQ

(2)

Từ (1) và (2) suy ra I, J, K, L, M, N, P, Q cùng nằm trên một đường tròn có

tâm là trung điểm của MP và NQ, gọi tâm đó là T. Hơn nữa, OM song song

HP( vì cùng vuông góc với AB), OP song song HM(vì cùng vuông góc với

CD) nên OMHP là hình bình hành, do đó trung điểm T của MP cũng là

trung điểm của OH.

Vậy I, J, K, L, M, N, P, Q cùng nằm trên một đường tròn có tâm là trung

điểm của OH.

b)(2 điểm) Ta sẽ dùng phép nghịch đảo để chứng minh phần này.

Kí hiệu (T) là đường tròn được nêu trong phần (a).Ta có

nên phép nghịch đảo cực H

phương tích biến I thành P, J thành Q, K thành M, L thành N.

Ta xét hai trường hợp:

+ Nếu IK và JL đều không đi qua H: thế thì phép nghịch đảo nêu trên biến

IK thành đường tròn (HPM), biến JL thành đường tròn (HQN). Gọi G là

giao điểm của IK và JL, phép nghịch đảo trên biến G thành G' thì G' thuộc

cả hai đường tròn (HPM) và (HQN). Vậy đường thẳng HG' là trục đẳng

phương của hai đường tròn này.

Ta có MNPQ là hình chữ nhật tâm T nên

do đó T thuộc trục đẳng phương HG'. Mà T là trung điểm của OH và H, G,

G' thẳng hàng nên G nằm trên đường đẳng OH.(đpcm)

+ Nếu IK hoặc JL đi qua H: giả sử IK đi qua H, thế thì AB song song CD

nên ABCD là hình thang nội tiếp đường tròn (O), do đó ABCD là hình thang

cân. Khi đó I, H, O, K thẳng hàng nên giao điểm của IK và JL nằm trên

đường thẳng OH.(đpcm)

  1. [SỞ GIÁO DỤC ĐÀO TẠO VĨNH LONG ĐỀ THI CHỌN HỌC SINH GIỎI VÒNG TỈNH LỚP 11 NĂM HỌC 2014 – 2015]

Cho tam giác nhọn nội tiếp đường tròn tâm bán kính , trên cạnh lấy các điểm sao cho điểmnằm giữa hai điểmvà . Gọilần lượt là hình chiếu vuông góc của trên các đường thẳng và , kéo dài cắt đường tròn ngoại tiếp tam giác tại. Chứng minh rằng tứ giác và tam giác có diện tích bằng nhau.

Hướng dẫn giải

N

M

D

O

A

B

C

E

F

Chứng minh rằng tứ giác AMDN và tam giác ABC có diện tích bằng nhau.

Đặt

(1)

Diện tích tứ giác AMDN là

=

. (2)

Vì tứ giác ABDC nội tiếp trong đường tròn nên ta có:

(3)

Từ (1), (2), (3) ta có điều phải chứng minh.

  1. [SỞ GIÁO DỤC ĐÀO TẠO VĨNH LONG ĐỀ THI CHỌN HỌC SINH GIỎI VÒNG TỈNH LỚP 11 NĂM HỌC 2015 – 2016]

Cho tam giác ngoại tiếp đường tròn tâm O. Chứng minh rằng , với .

Hướng dẫn giải

Gọi lần lượt là các tiếp điểm của với AB, AC, BC.

Ta có , nên

Tương tự: ,

Khi đó:

  1. [TRƯỜNG THPT CHUYÊN NGUYỄN TẤT THÀNH TỈNH YÊN BÁI NĂM 2015]

. Cho tam giác ABC. Gọi B­­­1 là điểm đối xứng của B qua AC, C1 là điểm đối xứng của C qua các đường thẳng AB, O1 là điểm đối xứng của O qua BC. Chứng minh rằng: Tâm đường tròn ngoại tiếp tam giác AB1C1 nằm trên đường thẳng AO1.

Hướng dẫn giải

Gọi H là trực tâm ΔABC. Gọi AB1, CH cắt nhau tại P, AC1 và BH cắt nhau tại Q. Gọi K là tâm đường tròn ngoại tiếp các tam giác AB1C1.

Dễ thấy O1 là tâm đường tròn ngoại tiếp tam giác HBC.

Xét tứ giác B1AHC. Ta có . Vì vậy B1AHC nội tiếp đường tròn (w1).Tương tự C1AHB nội tiếp đường tròn (w­2).

Trục đẳng phương của (K) và (w1) là AB1, Trục đẳng phương của (O1) và (w1) là CH. Nên P là tâm đẳng phương của (O1), (K) và (w1) và nó phải nằm trên trục đẳng phương của (O1) và (K).

Tương tự ta Chứng minh được Q nằm trên trục đẳng phương của các đường tròn (O1) và (K). Vì vậy, PQ vuông góc O1K.

Lại có ( cùng chắn cung AH của (w­2) nên PQB1C1 nội tiếp và tam giác AQP đồng dạng tam giác AC1B1.

Ta lại có nên KA là đường vuông góc PQ.

Vậy KA và O1K vuông góc PQ nên A, K, O1 thẳng hàng.

Bài 1. Cho tam giác nhọn với . Giả sử và là các điểm trên cạnh . sao cho và nằm giữa và . Giả sử là điểm thuộc miền trong tam giác sao cho và . Chứng minh rằng: .

Hướng dẫn giải

Vẽ hình bình hành , khi đó và giao nhau tại trung điểm của mỗi đường.

Do đó DE và PQ cũng giao nhau tại trung điểm M của mỗi đường suy ra PDQE là hình bình hành. Suy ra QE||PD từ đó A, E, Q thẳng hàng.

Vẽ hình bình hành BPAT. Khi đó ta cũng suy ra TACQ là hinh bình hành.

Ta có TQA=QAE=EAC=BAP=APT.

Do đó tứ giác TAQB nội tiếp.

Ta thấy qua phép tịnh tiến véc tơ thì tam giác BQT biến thành tam giác PCA.

Do đó ACB=TQB=TAB=ABP (ĐPCM).

Bài 2. Cho tam giác nhọn nội tiếp đường tròn tâm , là chân đường cao xuất phát từ và là trực tâm của tam giác Đường thẳng qua vuông góc với cắt cạnh tại Chứng minh rằng

Hướng dẫn giải

Gọi là giao điểm của và là giao điểm của và .

Theo định lý con bướm, suy ra là trung điểm của đoạn

Mặt khác là trung điểm của , do đó tứ giác là một hình bình hành. Suy ra

Mà do vậy

Bài 3. Cho tam giác nhọn có đường cao , trực tâm. Đường thẳng cắt đường tròn đường kính tại Đường thẳng cắt đường tròn đường kính tại Đường tròn ngoại tiếp tam giác cắt tại điểm thứ hai là .

a) Chứng minh rằng các điểm cùng thuộc một đường tròn.

b) Chứng minh rằng các đường thẳng đồng quy.

Hướng dẫn giải

a) Ta có Suy ra tứ giác GDFE nội tiếp. Dẫn đến (1).

Từ giả thiết ta có AB, AC lần lượt là đường trung trực của GF, DE. Suy ra A là tâm của đường tròn (GDFE). Suy ra (2).

Ta có (cùng chắn cung của đường tròn (DHF)). Xét đường tròn đường kính AB ta có (do (2)).

Suy ra (3).

Từ (1) và (3) suy ra Suy ra G, D, P thẳng hàng.

Tương tự E, F, P thẳng hàng.

A là điểm chính giữa cung của đường tròn đường kính AB nên Tương tự Suy ra

Suy ra các điểm G, H, P, E cùng thuộc một đường tròn.

b) Nhận thấy rằng, ba đường tròn (GDFE), (DHPF), (GHPE) có 3 trục đẳng phương DF, GE, HP đồng quy (tại Q). Từ đó, ta xét bài toán tổng quát như sau: “Cho tam giác PGE. Điểm Q thuộc tia đối của tia GE. Đường thẳng đi qua Q cắt cạnh PG, PE lần lượt tại D, F. Gọi K là giao điểm của GF DE. Đường thẳng PQ cắt EK, GK lần lượt tại B, C. Chứng minh rằng các đường thẳng BF, CD, PK đồng quy”.

Xét tam giác CQG, ta có (hàng điểm điều hoà cơ bản).

Suy ra (*).Áp dụng định lý Menelaus cho tam giác CBK với cát

tuyến PFE ta có (**)

Từ (*) và (**) suy ra Áp dụng định lý Ceva cho tam giác CBK suy ra BF, CD, PK đồng quy.

Bài 4. Cho hình bình hành . Các điểm nằm trên các cạnh sao cho . Gọi lần lượt là giao điểm của với . Chứng minh rằng diện tích tứ giác bằng diện tích tam giác .

Hướng dẫn giải

Đặt = k.Ta có

=

+) ,

+) Từ (1), (2), (3) suy ra

+) thay vào (1)

hay SΔAMN = 2 SΔAIJ, SIMNJ = SΔAMN - SΔAIJ = 2 SΔAIJ - SΔAIJ = SΔAIJ.

Bài 5. Cho và hai đường tròn tiếp xúc ngoài với nhau và tiếp xúc trong với . Gọi là tiếp điểm của và ; là tiếp điểm của với . Tiếp tuyến chung tại của cắt tại A. cắt tại ; cắt tại .

a) Chứng minh rằng .

b) cắt ở ; cắt tại . Chứng minh rằng là tâm đường tròn nội tiếp của tam giác .

c) Chứng minh rằng đồng quy.

Hướng dẫn giải

thuộc trục đẳng phương của và nên suy ra là tứ giác nội tiếp dẫn đến

----------------------------------------------------------------------------

b) Gọi là giao điểm của với .

Tam giác vuông tại có là đường cao

là tứ giác nội tiếp

------------------------------------------------------------------------------

Suy ra A là tâm đường tròn ngoại tiếp tam giác

Dẫn đến

Suy ra là phân giác của

Rõ ràng là phân giác của (do )

Vì thế là tâm đường tròn nội tiếp tam giác

-------------------------------------------------------------------------

c)Giả sử cắt tại , gọi là bán kính của , .

Rõ ràng là tâm vị tự ngoài của và ,lại có

Suy ra

Dẫn đến thẳng hàng (Menelauyt đảo)

Vậy đồng quy.

Bài 6. Cho hai đường tròn và cắt nhau tại . lần lượt là các đường kính của và . Gọi là trung điểm của ; là điểm thuộc đường phân giác của góc sao cho không vuông góc với và không thuộc hai đường tròn. Đường thẳng đi qua vuông góc với lần lượt cắt các đường tròn , tại các điểm khác . cắt đường tròn tại điểm thứ hai , cắt đường tròn tại điểm thứ hai .

1. Gọi là giao điểm của với . Chứng minh rằng là tiếp tuyến của đường tròn .

2. Chứng minh rằng 3 đường thẳng đồng quy.

Hướng dẫn giải

1. Không mất tính tổng quát giả sử là điểm thuộc đường phân giác trong của góc .

Ta có tứ giác là hình bình hành nên suy ra

Lại có

Do đó thẳng hàng. Chứng minh tương tự ta có thẳng hàng

Mặt khác

Do đó là tiếp tuyến của đường tròn

Nội dung

2. Ta có nên 4 điểm cùng thuộc đường tròn đường kính .

Mà nên suy ra là tiếp tuyến của đường tròn đường kính .

Do đó (1)

Mặt khác

(2)

Từ (1) và (2) suy ra

Vậy 4 điểm cùng thuộc một đường tròn.

Gọi là giao điểm của và

Vì 4 điểm cùng thuộc một đường tròn nên ta có

(3)

Ta có (4)

Gọi là giao điểm của với

Chứng minh tương tự câu 1) ta có là tiếp tuyến của đường tròn

Mặt khác tứ giác là hình thang vuông tại và là trung điểm của nên suy ra . Do đó (5)

Từ (3), (4), (5) suy ra cùng thuộc trục đẳng phương của hai đường tròn nên thẳng hàng. Vậy 3 đường thẳng đồng quy tại

*) Chú ý: Nếu HS không sử dụng góc định hướng thì phải xét các trường hợp vị trí của điểm ( nằm ngoài các đoạn nằm trong các đoạn )

Bài 1: (Kỳ thi học sinh giỏi tỉnh Hà Tĩnh 2008 – 2009) Trên mặt phẳng cho tứ giác lồi có .

a. Nếu biết . Hãy tính diện tích tứ giác ABCD theo .

b. Giả sử tứ giác thay đổi, mà không đổi. Hãy tìm giá trị lớn nhất của diện tích tứ giác

Bài 2: (Đề thi chọn HSG vòng tỉnh Vĩnh Long – NH: 2016 – 2017) Cho có 3 góc nhọn. Xác định điểm bên trong tam giác sao cho nhỏ nhất.

Hướng dẫn giải:

Dùng phép quay quanh A với góc quay biến M thành M’; C thành C’.

Ta có .

bé nhất khi bốn điểm B, M, M’, C’ thẳng hàng.

Khi đó . Ta được vị trí của M trong tam giác ABC.

Bài 3: (Đề thi chọn HSG tỉnh Vĩnh Long – NH : 2015 – 2016) Cho ngoại tiếp đường tròn tâm . Chứng minh rằng với , , .

Hướng dẫn giải

Gọi lần lượt là các tiếp điểm của với AB, AC, BC.

Ta có , nên

Tương tự: ,

Khi đó:

Bài 4: (Đề đề xuất thi chọn HSG khu vực duyên hải – đồng bằng Bắc bộ năm học 2015 – 2016, trường THPT chuyên Lương Văn Tụy) Cho đường tròn tâm đường kính , điểm trên tia đối của tia . Đường thẳng qua cắt đường tròn tại , . Hai tiếp tuyến của tại , cắt nhau tại , kẻ vuông góc với tại . Chứng minh rằng .

Hướng dẫn giải:

Gọi E, F thứ tự là giao của NI với (ω) suy ra CEDF là tứ giác điều hòa

ME, MF tiếp xúc với (ω).

Giả sử C nằm giữa M, D. Ta có tứ giác CDOI nội tiếp

Suy ra . Vậy ta có điều phải chứng minh.

V. Bài toán nội tiếp đường tròn

  1. (THPT Chuyên tỉnh Sơn La – Trại hè Hùng Vương lần X)

Cho đường tròn tâm và một dây cung không đi qua . là điểm chính giữa cung nhỏ , nằm ngoài đường tròn sao cho và nằm khác phía đối với đường thẳng . Qua kẻ tiếp tuyến với đường tròn , là tiếp điểm. cắt tại . Đường thẳng qua và vuông góc với cắt tại . Một đường thẳng thay đổi qua cắt dường tròn tại và ( nằm giữa và ), cắt tại .

Chứng minh rằng

1. Đường tròn tâm , bán kính tiếp xúc trong với đường tròn .

2. Tứ giác nội tiếp một đường tròn.

Lời giải

1) Tam giác OCT đồng dạng với tam giác IET vì:

Mà tam giác OCT cân tại O nên tam giác IET cân tại I, suy ra

Vậy: đường tròn tâm I bán kính IE tiếp xúc trong với đường tròn tâm (O) tại T.

2) Tam giác ACP đồng dạng với tam giác MCA vì

Do đó:

Tương tự, tam giác BCE đồng dạng với tam giác TCB vì

Do đó:

Suy ra . Vậy tứ giác PETM nội tiếp

  1. (THPT Chuyên Bắc Ninh – Tỉnh Bắc Ninh- Thi Olympic lần VII – 2013-2014)

Cho tam giác nội tiếp đường tròn tâm . Gọi là điểm chính giữa cung của đường tròn ; là trung điểm của ; , lần lượt là tâm đường tròn nội tiếp tam giác và . Chứng minh rằng , , cùng nằm trên một đường tròn.

Lời giải

và . Xét phép quay tâm M góc quay biến C thành N.

Gọi ảnh của điểm trong quép quay này là T, khi đó T nằm trên .

Ta định nghĩa tương tự điểm S trên .

Ta có , .

Do đó , suy ra đồng viên.

Ta có , tương tự .

Vì vậy . Mặt khác góc là góc ngoài tam giác , nên

.

Do đó nằm trên đường tròn đi qua các điểm .

Do đó ta suy ra ĐPCM

  1. (Trường THPT Chuên Vĩnh Phúc – Trại hè Hùng Vương lần X- 2014)

Cho tứ giác coa hai đường chéo cắt nhau tại . Một đường thẳng qua cắt đường tròn ngoại tiếp các tam giác , , , lần lượt tại , , , ( các điểm này khác ) sao cho là trung điểm của . Gọi là một điểm nằm trên đoạn ( khác , ). Gọi là giao điểm thứ hai của đường thẳng với đường tròn ngoại tiếp tam giác . Chứng minh rằng bốn điểm , , , cùng nằm trên một đường tròn.

Lời giải

Theo tính chất của hai đường tròn cắt nhau ta có:

Do đó tứ giác là hình bình hành suy ra cắt nhau tại điểm và là trung điểm của mỗi đoạn .

Bổ đề. Cho hai đường tròn cắt nhau tại hai điểm A, B. Gọi là trung điểm của . Một đường thẳng qua A và cắt lần lượt tại E, F. Khi đó khi và chỉ khi .

Thật vậy, gọi K, L lần lượt là trung điểm của AE, AF suy ra . Do đó tứ giác là hình thang vuông. Khi đó khi và chỉ khi là đường trung bình của hình thang hay . Vậy bổ đề được chứng minh.

Trở lại bài toán: Do O là trung điểm của MP nên theo bổ đề cho hai đường tròn và đường thẳng (d) ta được , cũng theo bổ đề trên cho và đường thẳng (d) ta được O là trung điểm của NQ. Từ đó suy ra (1).

Do bốn điểm M, R, N, S cùng nằm trên một đường tròn nên (2).

Từ (1) và (2) ta được bốn điểm P, Q, R, S cùng nằm trên một đường tròn.

  1. (THPT Chuyên Nguyễn Trãi – Hải Dương – Toán 11- 2015)

Cho tứ giác nội tiếp đường tròn tâm và có vuông góc với tại . Gọi

, , , lần lượt là hình chiếu của trên các đường thẳng , , , . Gọi , , , tương ứng là trung điểm của các cạnh , , , .

a. Chứng minh rằng các điểm , , , , , , , cùng nằm trên một đường tròn có tâm là trung điểm của đoạn thẳng .

b. Chứng minh giao điểm của nằm trên đường thẳng .

Lời giải

Trước hết ta chứng minh H, I, P thẳng hàng. Thật vậy, ( vì HP là trung tuyến của tam giác vuông HCD)

suy ra I, H, P thẳng hàng. Tương tự cũng có J, H, Q thẳng hàng, K, H, M

thẳng hàng, L, H, N thẳng hàng.

Rõ ràng I và K nằm trên đường tròn đường kính MP, J và L nằm trên đường

tròn đường kính NQ. (1)

Mặt khác, MNPQ là hình bình hành và MN song song AC, NP song song

BD, AC vuông góc BD nên MN vuông góc NP, vì vậy MNPQ là hình chữ

nhật. Do đó đường tròn đường kính MP cũng là đường tròn đường kính NQ

(2)

Từ (1) và (2) suy ra I, J, K, L, M, N, P, Q cùng nằm trên một đường tròn có

tâm là trung điểm của MP và NQ, gọi tâm đó là T. Hơn nữa, OM song song

HP( vì cùng vuông góc với AB), OP song song HM(vì cùng vuông góc với

CD) nên OMHP là hình bình hành, do đó trung điểm T của MP cũng là

trung điểm của OH.

Vậy I, J, K, L, M, N, P, Q cùng nằm trên một đường tròn có tâm là trung

điểm của OH.

b)(2 điểm) Ta sẽ dùng phép nghịch đảo để chứng minh phần này.

Kí hiệu (T) là đường tròn được nêu trong phần (a).Ta có

nên phép nghịch đảo cực H

phương tích biến I thành P, J thành Q, K thành M, L thành N.

Ta xét hai trường hợp:

+ Nếu IK và JL đều không đi qua H: thế thì phép nghịch đảo nêu trên biến

IK thành đường tròn (HPM), biến JL thành đường tròn (HQN). Gọi G là

giao điểm của IK và JL, phép nghịch đảo trên biến G thành G' thì G' thuộc

cả hai đường tròn (HPM) và (HQN). Vậy đường thẳng HG' là trục đẳng

phương của hai đường tròn này.

Ta có MNPQ là hình chữ nhật tâm T nên

do đó T thuộc trục đẳng phương HG'. Mà T là trung điểm của OH và H, G,

G' thẳng hàng nên G nằm trên đường đẳng OH.(đpcm)

+ Nếu IK hoặc JL đi qua H: giả sử IK đi qua H, thế thì AB song song CD

nên ABCD là hình thang nội tiếp đường tròn (O), do đó ABCD là hình thang

cân. Khi đó I, H, O, K thẳng hàng nên giao điểm của IK và JL nằm trên

đường thẳng OH.(đpcm)

LOẠI 2: Chứng minh các tính chất:tam giác, tứ giác đường tròn.

  1. Cho tam giác cân tại .Gọi là trung điểm . Đường tròn ngoại tiếp tam giác giao với phân giác góc tại nằm trong tam giác. Đường tròn ngoại tiếp tam giác giao với tại (khác ), giao với tại .giao với tại . Chứng minh rằng là tâm đường tròn nội tiếp tam giác .(Chuyên Hoàng Văn Thụ)

Hướng dẫn giải:

Gọi là trung điểm của và là trung điểm cạnh .

Ta có nằm trên đường tròn ngoại tiếp . Do tính đối xứng nên suy ra suy ra

Suy ra nằm trên phân giác góc hay là tia phân giác góc

Ta có:

suy ra cân tại và tam giác vuông tại .

Do nên thuộc trục đẳng phương của đường tròn đường kính và đường tròn ngoại tiếp .Từ đó đi qua giao điểm thứ hai của hai đường tròn này.

Ta có nên

Suy ra hay . Từ đó .Ta có (đpcm)

  1. Gọi là trung điểm cạnh của và lần lượt là hình chiếu của trên . Giả sử là giao điểm của các tiếp tuyến tại với đường tròn đường kính . Chứng minh rằng .(Tỉnh Nam Định 2010-2011-vòng 1)
  2. Gọi là giao điểm của hai đường chéo trong tứ giác lồi . Phân giác của góc cắt cạnh tại . Nếu , chứng minh rằng góc bằng góc .(Tỉnh Nam Định 2010-2011-vòng 2)

LOẠI 3: Tìm quỹ tích:

  1. Cho nửa đường tròn tâm đường kính và là một điểm chuyển động trên nửa đường tròn này. Gọi là chân đường vuông góc hạ từ xuống và là tâm đường tròn nội tiếp tam giác Tìm tập hợp các điểm (Trường THPT chuyên Hùng Vương Phú Thọ )

Phần thuận: Gọi là điểm chính giữa của cung . Xét trường hợp điểm chuyển động

trên cung nhỏ . Do là tâm đường tròn nội tiếp nên ta có:

Do (OA = OC, OI chung, ) nên suy ra được

Do cố định và luôn nhìn dưới một góc nên khi chuyển động trên cung thìchuyển động trên cung chứa góc dựng trên đoạn .

Phần đảo: Lấy điểm bất kì thuộc đoạn và vẽ đường thẳng vuông góc với tại cắt

nửa đường tròn tâm tại. Dựng phân giác trong của cắt cung chứa góc dựng

trên tại điểm . Ta chứng minh là tâm đường tròn nội tiếp của .

Do thuộc cung chứa góc 1350 dựng trên nên ngoài ra ta có (cgc) nên ,

Suy ra ⇒ tứ giác ACIH nội tiếp được tromg đường tròn

đườngkính Suy ra . Mặt khác ⇒ hay là tâm

đường trònnội tiếp của

Khi chuyển động trên cung nhỏ , chứng minh tương tự, cho chuyển động trên cung

chứa góc dựng trên đoạn .

Vậy khi chuyển động trên nửa đường tròn đường kính đã cho thì tâm đường tròn nội

tiếp của chuyển động trên 2 cung chứa góc dựng trên các đoạn thuộc nửa mặt phẳng bờ cùng phía với nửa đường tròn đã cho.

Loại 3: Tìm quỹ tích.

  1. [SỞ THỪA THIÊN HUẾ ( Vòng 1)- năm học 2002-2003]

a/ Trong mặt phẳng Oxy, cho một đường tròn (C) cắt parabol (P): y = x2 tại bốn điểm, một điểm có tọa độ là (1;1) và ba điểm còn lại là ba đỉnh của một tam giác đều. Tính bán kính của đường tròn (C).

b/ Tìm tập hợp các tâm của những tam giác đều có ba đỉnh thuộc parabol (P): y = x2.

Lời giải

a/ (C): (x – a)2 + (y – b)2 = R2. (C) qua điểm (1;1) nên: R2= (1 – a)2 + (1 – b)2.

Hoành độ x1, x2, x3 của ba đỉnh tam giác đều và x = 1 là nghiệm của phương trình:

(x – a)2 + (y – b)2 = (1 – a)2 + (1 – b)2

Do đó: x3 + x2 + (2 – 2b)x +2 – 2a – 2b ≡ (x – x­1)(x – x­2)(x – x­3) nên:

Từ giả thiết tam giác đều nên:

Do đó: a = , b = 3 và bán kính đường tròn (C) là: R = .

b/ (1.5 đ)

Thuận:

Giả sử I(a;b) là tâm của tam giác dều ABC có đỉnh trên (P). Đường tròn (ABC) cắt (P) thêm điểm M(x0;y0) ( có thể trùng A, B, C).

xA, xB, xC và x0 là nghiệm của: (x – a)2 + (y – b)2 = (x0 – a)2 + ( – b)2

Suy ra:

Hay: , vì vậy: b = 9a2 + 2. Nên tâm I ở trên đồ thị (G): y = 9x2 + 2.

Đảo: Xét I(a; 9a2 + 2) tùy ý trên (G): y = 9x2 + 2. Ta phải chứng minh đường tròn (C) tâm I bán kính IM với M(-3a; 9a2) cắt (P) thêm 3 điểm là 3 đỉnh của một tam giác đều.

Xét phương trình: (x – a)2 + (x2 – 9a2 – 2)2 = (-3a – a)2 + (9a2 – 9a2 – b)2 (2).

với f(x) = x3 -3a2x2 - (9a2 + 3)x + 27a3 + 7a (f(x) = 0 chính là phương trình (1) với x0 = - 3a).

Nếu a = 0: f(x) = x3 -3x = 0 có 3 nghiệm phân biệt.

Nếu a ≠ 0: Do f(x) liên tục, và

nên f(x) = 0 có 3 nghiệm phân biệt.

Vậy phương trình f(x) = 0 luôn có 3 nghiệm phân biệt x1, x2, x3 với mọi a.

y

x

A

B

C

M

I

Ta có: . Do đó:

Do x1, x2, x3 là 3 nghiệm phân biệt của (2) nên: (C) cắt (P) tại 3 đỉnh A, B, C của tam giác ABC. Và do (3): tam giác ABC có trọng tâm trùng với tâm đường tròn ngoại tiếp I, nên ABC là tam giác đều.

Kết luận: Tập hợp các tâm của những tam giác đều có 3 đỉnh thuộc (P): y = x2 là parabol:

(G): y = 9x2 + 2.

  1. [SỞ THỪA THIÊN HUẾ ( Vòng 1)- năm học 2004-2005]

Trong mặt phẳng , cho tam giác vuông cố định có , Tìm tập hợp các điểm thuộc mặt phẳng sao cho

Lời giải

Ta có:

+

+ Chọn hệ trục Axy và đơn vị trên trục sao cho B(3;0),C(0;3). Gọi M(x;y)

Vậy ở trong hình tròn (T) tâm I(-1;0), bán kính 2, ( kể cả biên).

Tương tự ở trong hình tròn (S) tâm J(0;-1), bán kính 2, ( kể cả biên).

Vậy tập hợp những điểm M thỏa bài toán là phần giao của hai hình tròn (T) và (S), ( kể cả biên).

y

x

  1. [SỞ THỪA THIÊN HUẾ ( Vòng 1)- năm học 2005-2006]

Cho hình vuông EFGH. Gọi (T) là đường tròn qua các trung điểm các cạnh của tam giác EFG. Nhận xét: Điểm H thỏa mãn đồng thời hai tính chất sau:

a/ Các hình chiếu vuông góc của nó lần lượt trên các đường thẳng: EF, FG, GE nằm trên một đường thẳng d.

b/ Đường thẳng d tiếp xúc với đường tròn (T).

Hãy tìm tập hợp tất cả các điểm N của mặt phẳng chứa hình vuông EFGH sao cho N thỏa mãn đồng thời hai tính chất a/ và b/ ở trên.

Lời giải

+ Điểm N thỏa mãn tính chất a/ khi và chỉ khi N ở trên đường tròn qua E, F, G.

+ Chứng minh: Chọn hệ trục Oxy với O là tâm của hình vuông EFGH và vec tơ đơn vị trên trục . Ta có E(-1;0), F(0;1), G(1;0).

Phương trình của EF: x –y + 1 = 0; FG: x + y -1 = 0, đường tròn (EFG): x2+y2=1

Gọi N(X;Y). Tọa độ các hình chiếu của N lên EG, EF, FG lần lượt là:

N1 (X;0), N2 ((X+Y-1); (X+Y+1)), N3 ((X-Y+1); (-X+Y+1))

;

thẳng hàng khi và chỉ khi (-X+Y-1)(-X)-(1-Y)(X+Y+1)=0X2+Y2=1(1)

+ Tìm thêm điều kiện để N thỏa mãn tính chất b/. Chỉ cần xét N(X;Y) khác F(0;1).

Với điều kiện (1) đường thẳng d có phương trình X(x-X) +(1-Y)(y-0)=0

Tâm của (T) là I(0; ). Bán kính của (T) là

+ d tiếp xúc (T) khi và chỉ khi

(2)

+ Giải hệ (1) và (2): Rút X2 từ (1) thay vào (2):

(2Y2-Y-1)2=2(1-Y)(Y-1)2(2Y+1) 2 =2(1-Y).Đang xét Y1 nên:(Y-1)(2Y+1)2= -2

4Y3-3Y+1= 0(Y+1)(4Y2-4Y+1) = 0 Y= -1; Y=.

+ VớiY=-1 ta cóđiểm N(0;-1),đó là H

Với Y= , ta có thêm hai điểm N là (;) và (-;)

Tập hợp phải tìm là ba đỉnh của tam giác đều nội tiếp trong đường tròn (EFGH) mà một đỉnh là H.

  1. [SỞ THỪA THIÊN HUẾ ( Bảng A- Vòng 1)- năm học 2000-2001]

Cho hình vuông cố định. Tìm tập hợp những điểm M trong hình vuông đó và thỏa mãn điều kiện: Tích hai khoảng cách từ điểm M đến hai cạnh của hình vuông cùng xuất phát từ một đỉnh bằng bình phương khoảng cách từ điểm M đến đường chéo của hình vuông không đi qua đỉnh đó.

A

B

C

D

M

N

Q

P

0

x

y

  1. (2.0 điểm)

Không giảm tính tổng quát, xét hình vuông có cạnh .

Đặt hình vuông ABCD lên mặt phẳng có hệ trục tọa độ

Oxy sao cho A(0;1), B(-1;0), C(0;-1), D(1;0).

Gọi M(x;y) là điểm ở trong hình vuông ABCD, hạ MN,

MP, MQ lần lượt vuông góc với BD, DA, AB tại

N, P, Q.

Do đó: MP.MQ = MN2 (1) ( xét 2 cạnh hình vuông phát xuất từ đỉnh A)

AB: x – y + 1 = 0,AD: x + y – 1 = 0.

(1)

M(x;y) ở trong hình vuông nên x – y + 1 > 0, và x + y – 1 < 0.

Do đó: x2 –(y – 1)2 = (x – y + 1)(x + y – 1) < 0 nên (1) ⇔ x2 – (y– 1)2 =- 2y2 ⇔ x2 + (y+1)2 = 2

Vậy tập hợp các điểm M là cung BD, cung ¼ đường tròn C, bán kính R = .

Từ kết quả trên ta kết luận: Tập hợp các điểm M là 4 cung ¼ đường tròn tâm là các đỉnh của hình vuông và có bán kính bằng cạnh của hình vuông.

  1. [Trường THPT DTNT Quế Phong- năm học 2008-2009]

Cho đường tròn (O) và điểm P nằm trong đường tròn đó. Một đường thẳng thay đổi đi qua P cắt (O) tại hai điểm A và B. Tìm quỹ tích điểm M sao cho .

LOẠI 3. Tìm quỹ tích:

  1. [ HỘI CÁC TRƯỜNG CHUYÊN VÙNG DUYÊN HẢI VÀ ĐỒNG BẰNG BẮC BỘ

TRƯỜNG THPT CHUYÊN LÀO CAI TỈNH LÀO CAI ]

Cho nhọn không cân, là một điểm chuyển động trên đoạn thẳng (nhưng không trùng vào các đầu mút). Đường tròn ngoại tiếp tam giác giao tại khác . Đường tròn ngoại tiếp tam giác giao tại Z khác . Gọi T là hình chiếu của trên BC, H là trực tâm tam giác . Gọi là giao của và . Gọi là điểm đối xứng với qua .

a) Chứng minh rằng thẳng hàng

b) Chứng minh rằng tâm của đường tròn ngoại tiếp tam giác luôn thuộc một đường thẳng cố định khi P thay đổi.

Hướng dẫn giải

Kí hiệu là đường tròn đi qua 3 điểm

a) Ta có .

Suy ra .

Suy ra hay .

Từ đó hay thẳng hàng

b) Gọi M là trung điểm BC, AM cắt đường tròn ngoại tiếp tam giác tại sao cho và khác phía với . AM cắt đường tròn ngoại tiếp tam giác tại khác A.

Do thuộc tròn ngoại tiếp tam giác nên

Suy ra .

Lại có và đối xứng nhau qua BC nên , .

Từ đó . Ta thu được

( vì bằng nửa tổng số đo cung A’B và KC của nhưng vì nên số đo cung A’B bằng số đo cung JC của. Do đó bằng nửa tổng số đo cung của , do đó bằng )

Vì vậy . Do

Vậy luôn đi qua giao điểm của trung tuyến ứng với đỉnh của hay hình chiếu của trực tâm trên .

Do đó tâm của đường tròn ngoại tiếp tam giác luôn thuộc đường thẳng trung trực của đoạn cố định khi thay đổi.

  1. [ Đề ôn thi đội tuyển Festival. Đề số 2 ]

Cho Đường thẳng và hai điểm cố định nằm trên , là điểm di động trên . Các đường tròn có tâm là và cùng đi qua cắt nhau tại ( khác ). Tìm tập hợp điểm

  1. Cho đường tròn và một điểm ở trong đường tròn. Xét tất cả các góc vuông đỉnh : gọi giao điểm của hai cạnh góc vuông với đường tròn là. Tìm tập hợp trung điểm của .

Hướng dẫn giải

Ta có

Mặt khác mà

Gọi là trung điểm , đặt (không đổi)

nên không đổi.

Vì cố định nên thuộc đường tròn tâm bán kính

Đảo lại, trên đường tròn lấy tuỳ ý . Lấy làm tâm quay một cung tròn bán kính cắt tại .

Ta có . Kéo dài cắt tại .

Xét tam giác có:

Hay tam giác vuông tại suy ra là trung điểm

nghĩa là hay tam giác vuông tại .

  1. [ THI HSG KHU VỰC DUYÊN HẢI & ĐỒNG BẰNG BẮC BỘ NĂM 2013 SỞ GD&ĐT QUẢNG NAM -TRƯỜNG THPT CHUYÊN NGUYỄN BỈNH KHI ÊM]

Cho đường tròn và một điểm cố định ở trong đường tròn (), đường thẳng qua vuông góc với cắt đường tròn tại và; là một điểm nằm trên đường tròn, tia đối xứng với tia qua đường thẳng cắt đường tròn tại . Gọi là trung điểm của .

a) Chứng minh đường thẳng đi qua một điểm cố định khi thay đổi trên đường tròn .

b) Gọi là giao điểm của đường thẳng với đường tròn. Đường thẳng và cắt nhau ở . Chứng minh rằng các điểm là những tâm của đường tròn nội tiếp và bàng tiếp của tam giác .

Hướng dẫn giải

Gọi là giao điểm của và ; là giao điểm của và.

Ta có và IE là phân giác của góc , suy ra:

Suy ra: (M là trung điểm của AB, New-tơn)

=

=

Mà ta lại có:

Do đó:

Suy ra:

Suy ra:

Suy ra . Suy ra . Vậy L cố định.

b)Trước hết ta chứng minh là phân giác của góc.

Gọi là giao điểm của OM với

Ta có:

Suy ra:

Suy ra:

Suy ra:

Ta có:

Do đó ta suy ra:

Suy ra:

Theo hệ thức Newton, ta suy ra: (1)

Mà nên là phân giác trong của góc (2)

Theo chứng minh trên ta có:

Suy ra:

Suy ra: (3)

Từ (1) và (3) ta suy ra: đồng quy tại .

Mà góc nên là tứ giác nội tiếp

Suy ra:

Suy ra là phân giác trong của góc . (4)

Từ (2) và (4), ta có là tâm đường tròn nội tiếp tam giác

Ta lại có suy ra là phân giác ngoài của góc. Suy ra là tâm đường tròn bàng tiếp của tam giác .

Vậy , lần lượt là tâm của đường tròn nội tiếp và bàng tiếp của tam giác .

  1. Xét các điểm ( không trùng với ) tương ứng thay đổi trên các đường thẳng chứa các cạnh của tam giác sao cho song song với và các đường thẳng cắt nhau tại . Gọi là giao điểm thứ hai (khác điểm ) của đường tròn ngoại tiếp các tam giác và .

1. Chứng minh rằng luôn nằm trên một đường thẳng cố định.

2. Gọi lần lượt là điểm đối xứng với qua các đường thẳng . Chứng minh rằng tâm đường tròn ngoại tiếp tam giác nằm trên một đường thẳng cố định.

Hướng dẫn giải

1) (2,0 điểm).

Do cùng nằm trên một đường tròn và cùng nằm trên một đường tròn, nên

Từ đó suy ra (2)

Gọi IJ theo thứ tự là hình chiếu của Q trên các đường thẳng BMCN. Khi đó, do (2) nên (do ).

Từ đó, theo tính chất của đường đối trung, Q nằm trên đường đối trung kẻ từ A của tam giác ABC.2) (2,0 điểm).

Gọi là giao điểm của AP với BC. Áp dụng định lý Céva cho tam giác ABC ta có

Do nên từ đó và (1) suy ra hay L là trung điểm BC. 0,5

Do AQ là đường đối trung nên và kết hợp với tứ giác nội tiếp nên suy ra (3). 0,5

Do cách xác định các điểm nên hay tam giác cân tại , kết hợp với là đường trung bình của tam giác

(4) 0,5

Từ (3), (4) suy ra là đường trung trực của đoạn B’C’ suy ra tâm đường tròn ngoại tiếp tam giác nằm trên đường thẳng AP hay nằm trên trung tuyến AL của tam giác ABC. 0,5

  1. Cho tam giác nhọn không cân tại là trung điểm cạnh và tương ứng là chân đường cao hạ từ của tam giác. là giao điểm của hai tiếp tuyến tại của đường tròn ngoại tiếp tam giác là giao điểm của và là giao điểm của và .

a) Chứng minh rằng là tâm đường tròn nội tiếp tam giác .

b) Đường tròn ngoại tiếp tam giác và cắt nhau tại điểm thứ hai . Chứng minh rằng trực tâm của tam giác nằm trên .

c) Chứng minh rằng nằm trên đường tròn ngoại tiếp tam giác .

Hướng dẫn giải

a) ZT là phân giác góc AZC.

Do góc XAB= goc ACB = góc BFE =góc AFX và TA= TF, từ đó X và T nằm trên trung trực của AF, do đó T là tâm đường tròn nội tiếp tam giác XYZ

b) Giả sử AB < BC, khi đó D nằm trên cung nhỏ AB. Gọi O là tâm đường tròn ngoại tiếp tam giác ABC và L là trung điểm của BH. Ta có được BD và LO vuông góc.

Từ BD và DH vuông góc, ta được LO và DH song song. OLHT là hình bình hành nên OL song song với HT, do đó D, H, T thẳng hàng.

c) Chứng minh được góc ADT = góc AXT và TY là đường trung trực của DC.

Chứng minh được góc CDT = góc CYT nên CTDY là tứ giác nội tiếp.

Do đó góc XDY + góc XZY= góc XDT+ góc TDY+ góc XZY

=góc ZAT + góc ZCT + góc XZY = 1800, do đó DXZY là tứ giác nội tiếp.

  1. (Đề thi đề xuất trường PT vùng cao Việt Bắc – 2015) Từ một điểm cố định ta vẽ hai tiếp tuyến đến những đường tròn thay đổi tâm sao cho hai tiếp tuyến đó luôn vuông góc với nhau.

a. Tìm tập hợp tâm của những đường tròn đi qua một điểm cố định khác với .

b. Cho đường tròn có tâm chạy trên một đường thẳng cố định không đi qua . Tìm tập hợp các tiếp điểm và của nhữn đường tròn đó với các tiếp tuyến vẽ từ .

Hướng dẫn giải:

a. Tứ giác OTCT’ có 3 góc vuông và nên nó là một hình vuông. Gọi R là bán kính của đường tròn (C), ta có .

Do đó: và

Vậy tâm C ở trên đường tròn tâm I là tập hợp những điểm có tỉ số khoảng cách tới AO bằng . Đường kính DE của đường tròn tâm I đi qua các điểm AO tạo nên một hàng điểm điều hòa; ta có

Ngược lại lấy điểm C’ bất kỳ trên đường tròn tâm I, ta có .

Từ O kẻ hai tiếp tuyến OT1 và ta có . Vậy là hình vuông.

Vậy tập hợp các điểm C là đường tròn tâm I với I là trung điểm của đoạn DE trong đó D, E, O, A là một hàng điểm điều hòa.

b. và .Vậy T là ảnh của C trong phép đồng dạng tâm O tỉ số , góc quay .

Điểm C chạy trên đường thẳng nên điểm T chạy trên đường thẳng là ảnh của trong phép đồng dạng trên.

Với điểm T’ ta dùng phép đồng dạng tâm O tỉ số , góc quay ta tìm được tập hợp các điểm T’ là đường thẳng ảnh của trong phép đồng dạng .

LOẠI 3: Tìm quỹ tích:

Câu TRƯỜNG THPT: LÊ QUÝ ĐÔN

Trong mặt phẳng cho tam giác đều cạnh, hai tia vuông góc với và ở cùng một phía đối với. Hai điểm lần lượt chuyển động trên và.

1/ Gọi là trung điểm của là hình chiếu của B trên mặt phẳng

Góc giữa và bằng . Tính độ dài đoạn theo a và

2/ Gọi là mặt phẳng qua B và vuông góc với. Chứng minh rằng

luôn đi qua một đường thẳng cố định.

3/ Gọi O là trung điểm, không đổi, kẻ vuông góc với tại. Chứng minh rằng chạy trên một đường tròn cố định.

Câu SỞ GIÁO DỤC VÀ ĐÀO TẠO LONG AN CẤP TỈNH VÒNG 2

Cho tam giác . Gọi là điểm chuyển động trên cạnh . Gọi là điểm chuyển động trên cạnh .

a) Giả sử .Chứng minh đường trung trực của luôn đi qua một điểm cố định.

b) Giả sử không đổi.Chứng minh luôn đi qua một điểm cố định

Hướng dẫn giải

a) Nếu tam giác cân thì trung trực đi qua điểm cố địnhXét tam giác không cân tại A

Gọi E là điểm chính giữa cung của đường tròn ngọai tiếp tam giác

ABC.E là điểm cố địnhvì ;góc =góc nên Suy ra: hay đường trung trực của luôn đi qua điểm E cố định

b) Kẻ đường phân giác trong của cắt MN tại FGọi là số đo góc BACTa có: diện tích =diện tích +diện tích Suy ra:

không đổi hay F là điểm cố định

Vậy luôn đi qua một điểm cố đinh.

Câu SỞ GIÁO DỤC VÀ ĐÀO TẠO LONG AN (VÒNG 1)

a.Cho tam giác vuông cân tại, cạnh.Trong mặt phẳng chứa tam giác lấy thỏa .Tìm quĩ tích điểm

b.Cho tam giác có hai trung tuyến và hợp với nhau một góc, ,. Tính độ dài trung tuyến còn lại của tam giác.

Hướng dẫn giải

• Chọn hệ trục tọa độ vuông góc sao cho tia qua A và tia qua. Ta có: , , . Giả sử .

a. • Chọn hệ trục tọa độ vuông góc sao cho tia qua A và tia qua . Ta có: , , . Giả sử .

. • Phương trình trên là phương trình của một đường tròn tâm , bán kính . • Vậy quỹ tích điểm M là một đường tròn tâm , bán kính .

b. Gọi G là trọng tâm của tam giác.

• Xét trường hợp:

Ta có:

Vậy AC2 = 112 Vậy AB2 = 52 Vậy độ dài trung tuyến còn lại:

Xét trường hợp:

Ta có:

Vậy AC2 = 208 Vậy AB2 = 148

Vậy độ dài trung tuyến còn lại:

Câu KỲ THI HỌC SINH GIỎI CẤP TỈNH LỚP 12

LONG AN VÒNG 2-NĂM 2013

Trong mặt phẳng cho đường tròn tâm bán kính và điểm cố định thuộc đường tròn . Gọi là tiếp tuyến của tại điểm. Tìm quỹ tích điểm M biết rằng khoảng cách từ M đến đường thẳng Δ bằng độ dài tiếp tuyến MT của đường tròn với T là tiếp điểm.

Hướng dẫn giải

A

M

T

H

x

y

I

Δ

Chọn hệ tọa độ Oxy như hình vẽ.

Khi đó ta có Gọi M(x; y)

Tam giác MTI vuông tại T

Thử lại: Gọi

Vậy quỹ tích điểm M là một parabol

Câu KỲ THI CHỌN HỌC SINH GIỎI CẤP TỈNH LỚP 12

LONG AN VÒNG 2 NĂM 2015

Cho tứ giác lồi có hai đường chéo không vuông góc với nhau, nội tiếp đường tròn . Gọi là điểm di chuyển trên cung không chứa . Gọi là giao điểm của với , là giao điểm của với . Đường tròn ngoại tiếp các tam giác và cắt nhau tại giao điểm thứ hai . Chứng minh rằng luôn đi qua một điểm cố định.

Hướng dẫn giải

Gọi lần lượt là tiếp tuyến của tại . Khi đó:

. Do đócố định Tương tự cố định.

Gọi , suy ra cố định. Ta có: Khi đó tam giáccân tại nên hay K thuộc trục đẳng phương của hai đường tròn và .

Vậy đi qua điểm cố định.

Câu LONG AN VÒNG 2 - NĂM 2012

Cho đường tròn có tâm là và đường kính là, là điểm cố định nằm giữa và . Gọi là đường thẳng qua và cắt tại và .

a) Tìm điểm M trên sao cho .

b) Gọi đối xứng qua và giả sử thay đổi nhưng luôn qua . Chứng minh: luôn nhận giá trị không đổi.

Hướng dẫn giải

a) Ta có: ( là trung điểm )

Th1: OI. Khi đó mọi M nằm trên là điểm M cần tìm. Th2: OI.Khi đó Vậy điểm cần tìm là giao điểm của đường thẳng với

với là đường thẳng qua và vuông góc . b) Xét ta có: Tương tự xét : Mặt khác: Thay thế các đẳng thức vào ta được : không đổi

Loại 4: Bài toán dựng hình.

  1. [SỞ THỪA THIÊN HUẾ ( Bảng A-Vòng 2)- năm học 1998-1999]

Trong mặt phẳng cho một đường tròn (C), giả sử tâm của nó chưa được đánh dấu. A là một điểm trong mặt phẳng. Chỉ dùng thước thẳng hãy dựng qua A tiếp tuyến của đường tròn (C). ( Thước thẳng là dụng cụ để vẽ đường thẳng).

+ Bổ đề 1:

Từ một điểm M ngoài đường tròn (C) tâm O bán kính R vẽ hai tiếp tuyến MA, MB. Hai cát tuyến đi qua M cắt đường tròn PQ, RS thì giao điểm hai đường thẳng SP và QR thuộc đường thẳng AB

_

K

_

O

_

X

_

Y

_

I

_

J

_

R

_

M

_

A

_

B

_

Q

_

S

_

P

+ Bổ đề 2: X là một điểm cho trước, tập hợp tất cả các điểm Y sao cho là đường thẳng dx vuông góc với Ox. X, X’, X” thẳng hàng và O không thuộc XX’ thì dx, dx’, dx” đồng quy.

Trở lại bài toán:

a/ Nếu A không thuộc đường tròn thì sử dụng bổ đề 1.

b/ Nếu A thuộc đường tròn. Chọn B, C sao cho A,B,C thẳng hàng và B,C ở ngoài đường tròn. Dùng bổ đề 1 và 2, dựng tiếp tuyến BE, BF và CG, CH, EF và GH cắt nhau tại T thì AT là tiếp tuyến cần dựng.

Loại 5: Hình học Oxy về điểm.

  1. [SỞ THỪA THIÊN HUẾ ( Vòng 1)- năm học 2003-2004]

Tìm hai điểm A, B lần lượt ở trên elip (E) và đường tròn (C):

, (C): (x – 11)2 + (y – 13)2 = 34.

sao cho độ dài AB là nhỏ nhất.

Lời giải

(C) là đường tròn tâm I(11;13) bán kính R =.

Nhận xét rằng A∈(E), B∈(C) nên đoạn AB ngắn nhất thì ba điểm I, A, B thẳng hàng.

A(x0;y0) ∈ nên

IA2=(x0–11)2+(y0 – 13)2=.

IA2=290+50cos2t+18sin2t -110cost - 78sint.

.

Dấu bằng xảy ra khi chỉ khi: t = ⇔ A(5;3).

Vậy độ dài AB nhỏ nhất là: d =2-= khi A(5;3) và từ đó suy ra được B(8;8).

y

x

I

B

A

  1. [Đề HSG 11-Bảng A]

Trong mặt phẳng tọa độ Oxy, cho tam giác ABC có đường tròn nội tiếp tiếp xúc với ba cạnh BC, CA, AB lần lượt tại M, N, P. Gọi D là trung điểm của cạnh BC. Biết M(-1;1), phương trình NP là x+y-4=0 và phương trình AD là 14x-13y+7=0. Tìm tọa độ điểm A.

Lời giải

B

C

A

I

N

M

K

D

P

E

F

Kéo dài IM cắt NP tại K. Kẻ đường thẳng qua K

song song với BC cắt AB, AC lần lượt tại E, F.

Ta có: các tứ giác KEPI và KNFI nội tiếp nên

Mà suy ra

Do đó, K là trung điểm EF

Suy ra A, K, D thẳng hàng

hay K là giao điểm của NP và AD

Tọa độ K là nghiệm của hệ

Phương trình IM đi qua M và K là

Ta có:

Vì I và M cùng phía với NP nên ta có I(1;2). Khi đó A(6;7)

  1. [Trường THPT Đô Lương 3- Nghệ An- năm học 2012-2013]

Trong mặt phẳng tọa độ Oxy, cho tam giác ABC và đường thẳng (d): x-y+1=0. Gọi D(4;2), E(1;1), N(3;0) lần lượt là chân đường cao kẻ từ A, chân đường cao kẻ từ B và trung điểm cạnh AB. Tìm tọa độ các đỉnh của tam giác ABC, biết rằng trung điểm M của cạnh BC nằm trên đường thẳng (d) và điểm M có hoành độ lớn hơn 3.

LOẠI 5:Hình học Oxy về điểm

  1. [CHỌN HSG NĂM HỌC 2015-2016-VĨNH PHÚC]

Trong mặt phẳng với hệ tọa độ , cho tam giác cân tại , là trung điểm của . Đường thẳng và là trọng tâm tam giác . Đường thẳng đi qua điểm . Tìm tọa độ các đỉnh của tam giác , biết điểm có hoành độ dương và tâmđường tròn ngoại tiếp tam giác thuộc đường thẳng

Hướng dẫn giải

Gọi I là tâm đường tròn ngoại tiếp tam giác ABC. Trước hết ta chứng minh Thật vậy, gọi lần lượt là trung điểm . Suy ra là trọng tâm tam giác Mặt khác là trọng tâm tam giác nên . Suy ra . Mà nên .

Rõ ràng nên là trực tâm tam giác . Suy ra .

Đường thẳng qua và vuông góc với nên có phương trình:

Tọa độ thỏa mãn hệ

Gọi Ta có

Suy ra , . Từ đó suy ra Gọi .

Do là trọng tâm ACM nên. Mà suy ra

Từ đó Thử lại ta thấy. Suy ra không tồn tại

LOẠI 5:Hình học Oxy về điểm.

  1. Trong mặt phẳng tọa độ Oxy, cho hình bình hành có . là điểm nằm bên trong hình bình hành sao cho và . Tìm tọa độ điểm biết .(Cụm Quỳnh Lưu 2016-2017)

Hướng dẫn giải:

Gọi là điểm thứ tư của hình bình hành MABE, dễ thấy MECD cũng là hình bình hành nên

Mà suy ra hay tứ giác BECM nội tiếp.

Suy ra

Ta có hay vuông tại M

Vì .

Ta có .

Giả sử ta có .

Giải hệ phương trình trên được hai nghiệm:

Vậy có hai điểm thỏa mãn đề bài là:

  1. Trong mặt phẳng , cho có và tâm đường tròn nội tiếp là . Gọi lần lượt là tiếp điểm của đường tròn với các cạnh . Điểm là giao điểm của với . Tìm tọa độ các đỉnh của tam giác biết phương trình (Cụm Quỳnh Lưu –Hoàng Mai 2016-2017)

Hướng dẫn giải:

Ta có:

tứ giác nội tiếp, mà tứ giác nội tiếp đường tròn đường kính nên

thuộc đường tròn đường kính

+) Viết được phương trình

+) Lấy đối xứng qua

+) Viết được phương trình

  1. [ THI CHỌN HỌC SINH GIỎI TỈNH NGH Ệ AN- 2015-2016 ]

Trong mặt phẳng tọa độ , cho tam giác có đường tròn nội tiếp tiếp xúc với ba cạnh lần lượt tại . Gọi D là trung điểm của cạnh . Biết , phương trình là và phương trình là . Tìm tọa độ điểm .

  1. (THPT Diễn Châu 2 –Nghệ An- thi học sinh giỏi trường 2016-2017 toán 10).

Trong mặt phẳng tọa độ cho hai đường thẳng và điểm .

a. Lập phương trình đường thẳng đi qua cắt tại và cắt tại sao cho: .

b. Lập phương trình đường thẳng đi qua và tạo với , một tam giác cân đỉnh .

  1. (THPT Diễn Châu 2 –Nghệ An- thi học sinh giỏi trường 2016-2017 toán 10).

Cho đường tròn và đường thẳng . Đường thẳng cắt đường tròn tại hai điểm , .

a. Tìm tọa độ , và tính độ dài .

b. Tìm điểm thuộc đường tròn sao cho diện tích tam giác lớn nhất.

  1. ( Sở GDĐT Nghệ An- thi chọn học sinh giỏi tỉnh 2005-2006 lớp 12)

Trong mặt phẳng tọa dộ cho hình chữ nhật tâm . Phương trình đường thẳng là và . Tìm tọa độ các đỉnh, , , biết rằng đỉnh có hoành độ âm.

  1. (Đề thi chọn học sinh giỏi 11)

Cho tam giác có , , và các điểm trên cạnh , trên cạnh , và trên cạnh sao cho là hình vuông. Tìm tọa độ các điểm , , , .

  1. (Đề thi chọn học sinh giỏi 11)

Cho hình chữ nhật có diện tích là . Phương trình : , điểm thuộc đường thẳng , điểm thuộc đường thẳng . Viết phương trình đường thẳng chứa các cạnh của hình chữ nhật biết điểm có hoành độ là một số nguyên.

Lời giải

Gọi B(b; 2b – 3),

Do b nguyên nên b = 1

Vậy B(1; - 1) PT AB: 3x + 4y + 1 = 0, PT BC: 4x – 3y – 7 = 0

Loại 6: Hình học Oxy về đường thẳng.

  1. [Đề chọn HSG lớp 11]

Cho đường tròn (C): x2+y2=R2 và điểm M (a,b) nằm ngoài đường tròn. Từ M kẻ hai tiếp tuyến MT1 và MT2 đến đường tròn (T1, T2 là các tiếp điểm). Viết phương trình đường thẳng T1T2.

  1. [ Đề ôn thi đội tuyển festival. Đề số 3 ]

Viết phương trình các cạnh của tam giác, biết, đường cao và phân giác trong qua đỉnh lần lượt có phương trình là và.

LOẠI 6:Hình học Oxy về đường thẳng.

  1. [SỞ GIÁO DỤC ĐÀO TẠO VĨNH LONG ĐỀ THI CHỌN HỌC SINH GIỎI VÒNG TỈNH LỚP 11 NĂM HỌC 2015 – 2016]

Cho tam giác có đỉnh , đường trung tuyến có phương trình và phân giác trong có phương trình . Viết phương trình đường thẳng .

Hướng dẫn giải

Ta có:

Trung điểm M của AC là

Ta có . Vậy .

Từ kẻ AK vuông góc CD tại I .

Phương trình đường thẳng AK: .

Tọa độ điểm I:

Ta có tam giác ACK cân tại C nên I là trung điểm AK

Phương trình đường thẳng đi qua C

LOẠI 7:Hình học Oxy về đường tròn.

  1. [SỞ GIÁO DỤC ĐÀO TẠO VĨNH LONG ĐỀ THI CHỌN HỌC SINH GIỎI VÒNG TỈNH LỚP 11 NĂM HỌC 2014 – 2015]

Trong mặt phẳng, với hệ toạ độ cho đường tròn (C1): , đường tròn (C2): . Gọi giao điểm có tung độ dương của (C1) và (C2) là , viết phương trình đường thẳng đi qua cắt (C1) và (C2) theo hai dây cung có độ dài bằng nhau.

Hướng dẫn giải

(C1) có tâm , bán kính

(C2) có tâm , bán kính

Giao điểm của (C1) và (C2) là và (Vì A có tung độ dương nên A(2;3).

Đường thẳng d qua A có phương trình: hay .

Gọi

Yêu cầu bài toán trở thành:

Với , chọn , suy ra phương trình d là:

Với , chọn , suy ra phương trình d là: .

Bài 1: (ĐỀ THI HSG – THPT Dương Xá – NH: 2008 – 2009)

Cho họ đường thẳng . Tìm các điểm trên mặt phẳng tọa độ sao cho không có bất kỳ đường thẳng nào thuộc họ đi qua.

Hướng dẫn giải:

Gọi là điểm cần tìm, khi đó phương trình sau:

vô nghiệm

TH1: . luôn có nghiệm

TH2: , khi đó (1) vô nghiệm

(I)hoặc (II)

Từ đó suy ra các điểm thỏa mãn là phần không bị gạch trong hình nhưng không bao gồm cạnh và không bao gồm đỉnh

Bài 2: (ĐỀ THI HSG – THPT Dương Xá – NH: 2008 – 2009) Cho vuông tại có hai đường trung tuyến . Gọi là góc giữa hai đường thẳng . Chứng minh rằng khi đó .

Hướng dẫn giải:

Chọn hệ trục tọa độ như hình vẽ:

; ;

;

Áp dụng bất đẳng thức côsi ta có

Suy ra . Dấu bằng xảy ra khi và chỉ khi

Bài 3: (Kỳ thi HSG cấp tỉnh Trà Vinh năm học 2014 – 2015) Trong mặt phẳng Oxy, cho điểm . Viết phương trình đường thẳng qua E và cắt hai trục , tại hai điểm sao cho:

a. có chu vi nhỏ nhất.

b. Khoảng cách từ đến lớn nhất.

Bài 4: (Đề thi chọn HSG tỉnh Vĩnh Long – NH : 2015 – 2016) Cho có đỉnh , đường trung tuyến có phương trình và phân giác trong có phương trình . Viết phương trình đường thẳng .

Hướng dẫn giải:

Ta có: . Trung điểm M của AC

Ta có . Vậy .

Từ kẻ AK vuông góc CD tại I .

Phương trình đường thẳng AK: .

Tọa độ điểm I:

Ta có tam giác ACK cân tại C nên I là trung điểm AK

Phương trình đường thẳng đi qua CK là:

Bài 5: (Đề thi đề nghị trường THPT chuyên Lê Quý Đôn TP. Đà Nẵng – hội thi HSG duyên hải Bắc bộ lần thứ VII) Cho -giác đều nội tiếp đường tròn và đường thẳng tùy ý. Qua các điểm vẽ các đường thẳng song song với cắt đường tròn tại các điểm . Chứng minh tổng không phụ thuộc vào vị trí của đường thẳng .

Hướng dẫn giải:

Chọn hệ trục , sao cho gốc tọa độ là tâm đa giác, trục vuông góc với . Không mất tính tổng quát, giả sử có thể giả sử đa giác đều nội tiếp đường tròn đơn vị (1).

Đặt thì

và ,.

Vậy

  1. [ĐỀ THI HSG TỈNH NGHỆ AN NĂM 2013-2014] Trong mặt phẳng toạ độ , cho hình vuông có . Biết thuộc cạnh và thuộc đường thẳng . Tính diện tích hình vuông .

LOẠI 6:Hình học Oxy về đường thẳng.

  1. Trong hệ tọa độ , cho đường tròn (): và điểm . Chứng minh rằng nằm trong đường tròn, hãy viết phương trình đường thẳng qua cắt đường tròn () tại hai điểm sao cho .

(Quảng Xương II)

Hướng dẫn giải:

Tâm bán kính

Ta có suy ra nằm trong đường tròn

Gọi là trung điểm suy ra , ta tính được

Suy ra đường thẳng cần tìm qua và khoảng cách từ tới đt cần tìm bằng 1.Ph. trình đt có dạng:

Ta có

Tìm được 2 đt là: và

  1. a) Viết PT đường thẳng đi qua giao điểm của hai đường thẳng

và đồng thời chắn trên hai trục tọa độ những đoạn bằng nhau.

b) Cho , biết . Viết phương trình đường phân giác trong của đỉnh (Trường THPT Kim Bôi)

Loại 7: Hình học Oxy về đường tròn.

  1. [SỞ THỪA THIÊN HUẾ ( Bảng B- Vòng 2)- năm học 1999-2000]

Cho parabol (P): y2=2x và đường tròn (C):x2+y2–8x+12=0.

Chứng minh rằng có vô số tam giác với ba đỉnh trên (P) mà các cạnh tiếp xúc với (C).

Lời giải

Đường tròn (C) có tâm I(4,0), bán kính R = 2.

Lấy A(x1; y1), B(x2; y2) tùy ý ( y1≠ y2) thuộc (P), phương trình đường thẳng AB là:

AB: (y - y1)(x2 - x1) = (y2 - y1)(x - x1)

Do A, B ∈ (P) nên , do đó: AB: 2x – (y1 + y2)y + y1.y2 = 0.

Tìm điều kiện tiếp xúc:

AB tiếp xúc (C)⇔.

Tượng tự, nếu C(x3; y3) thuộc (P) và y1 ≠ y3, ta có:

AC tiếp xúc (C).

Do đó nếu AB và AC tiếp xúc (C) ta được (1) và (2). Điều này chứng tỏ y1 và y3 là hai nghiệm của phương trình ẩn y:

Với y1 ≠ ± 2, (3) là phương trình bậc hai có Δ’ > 0 nên (3) luôn có hai nghiệm y2 và y3:

Do đó, thế vào ta được: .Vậy theo điều kiện tiếp xúc ta được BC tiếp xúc (C). Và từ các kết quả trên chứng tỏ rằng có vô số tam giác thỏa đề bài.

  1. [Trường THPT Quế Võ 1- năm học 2008-2009- Bắc Ninh]

Lập phương trình đường tròn ( C ) qua điểm A(-1; -2) và tiếp xúc với đường thẳng d: tại điểm M(1; 2).

Lời giải

Viết được pt đường thẳng đi qua tâm I của đường tròn (C) và vuông góc với đường thẳng d là từ đó suy ra I(1+7t;2-t)

+) (C) tiếp xúc với d khi và chỉ khi IM=R IM2=R2 R2=50t2

+) (C) có dạng (x-1-7t)2+(y-2+t)2=50t2

+) A (C) t=-1. Vậy (C): (x+6)2+(y-3)2=50.

  1. (Đề thi chọn HSG vòng tỉnh Vĩnh Long – NH: 2016 – 2017) Trong mặt phẳng cho có đỉnh , trọng tâm và trực tâm . Tìm tọa độ của và tính bán kính của đường tròn ngoại tiếp .

Hướng dẫn giải:

Gọi M là trung điểm cạnh BC, ta có

hay là pháp vectơ của đường thẳng BC.

Phương trình

B C đối xứng với nhau qua M nên gọi thì có .

; . Ta có:

. Vậy có hoặc

Kẻ đường kính AK của đường tròn (C) ngoại tiếp tam giác ABC

Tứ giác BHCK có BH//KCBK//HC nên BHCK là hình bình hành. Suy ra: HKBC cắt nhau tại M là trung điểm của BCM cũng là trung điểm của HK.

Ta có ,. Bán kính

LOẠI 7:Hình học Oxy về đường tròn.

  1. Trong mặt phẳng toạ độ, viết phương trình đường tròn () là ảnh của của đường tròn () có phương trình qua phép đối xứng trục Đ,

với (Trường THPT Quế Võ )

LOẠI 8: Các bài toán khác

  1. Cho . Phân giác trong của các góc cắt đường tròn ngoại tiếp lần lượt tại các điểm. Đường thẳng cắt đường thẳng tại điểm ; đường thẳng cắt đường thẳng tại điểm ; đường thẳng cắt đường thẳng tại điểm . Gọi O là tâm đường tròn ngoại tiếp tam giác . Đường thẳng cắt đường thẳng tại điểm M. Biết rằng và . Tính các góc của tam giác ABC. (chuyên Vĩnh Phúc)

HDG:* Dễ thấy , do đó O là trung điểm của .

*

* Do BM=MN; Do đó , mà

Vậy

Cùng với ta được

Loại 8: Các bài toán khác.

  1. [SỞ THỪA THIÊN HUẾ ( Bảng A-Vòng 1)- năm học 1999-2000]

Tập hợp M gồm hữu hạn điểm trên mặt phẳng sao cho với mọi điểm X thuộc M tồn tại đúng 4 điểm thuộc M có khoảng cách đến X bằng 1. Hỏi tập hợp Mcó thể chứa ít nhất là bao nhiêu phần tử?.

Lời giải

Rõ ràng có ít nhất hai điểm P,Q thuộc M sao cho PQ ≠ 1.

Ký hiệu MP={X∈M/PX=1}. Từ giả thiết |MP|=4 ta có: |Mp∩Mq|≤2.

Nếu tồn tại P, Q sao cho |Mp∩Mq|≤1 thì M chứa ít nhất 9 điểm.

Trường hợp với mọi P,Q sao cho PQ ≠ 1 và |Mp ∩ Mq| = 2.

Khi đó Mp∩Mq={R,S}, lúc đó MP={R,S,T,U} và Mq={R,S,V,W} và giả sử

M = {P,Q,R,S,T,U,V,W} ta có TQ ≠ 1, UQ ≠ 1, VP ≠ 1, WP ≠ 1.

Nếu TR,TS,UR,US khác 1: suy ra Mt∩Mq=Mu∩Mq={V,W} suy ra T hay U trùng với Q, vô lý.

Nếu TR,TS,UR,US có một số bằng 1: Không giảm đi tính tổng quát, giả sử TV = 1 lúc đó TS ≠ 1 và TV=1 hay TW=1. Giả sử TV=1 lúc đó TW≠1 suy ra TU = 1, và Mt = {P,R,U,V} và Mu={P,T,V,W} lúc đó UTV, RPT,UTV là các tam giác đều cạnh 1, ta có hình 1. Điều này mâu thuẫn vì VR>2.

Vậy M chứa ít nhất là 9 điểm. Dấu bằng xảy ra với hình2.

Vậy M có thể chứa ít nhất là 9 điểm.

A4

A8

A6

A5

A9

A7

A1 A2

A3

V

T

R

U

P

  1. [Trường THPT Trần Nguyên Hãn- Hải Phòng- năm học 2008-2009]

Cho tam giác đều ABC:

M là điểm nằm trong tam giác sao cho . Hãy tính góc

Một điểm S nằm ngoài mặt phẳng (ABC) sao cho tứ diện SABC đều, gọi I, K là trung điểm của các cạnh AC và SB. Trên đường thấng AS và CK ta chọn các điểm P, Q sao cho PQ // BI. Tính độ dài PQ biết cạnh của tứ diện có độ dài bằng 1.

  1. [Trường THPT Trần Nguyên Hãn- Hải Phòng- năm học 2008-2009]

Cho tam giác ABC có 3 góc nhọn. Xác định điểm M bên trong tam giác sao cho

MA+MB+MC nhỏ nhất.

Lời giải

Dùng phép quay quanh A với góc quay 600 biến M thành M’; C thành C’

Ta có MA+MB+MC = BM+MM’+M’C’

MA+MB+MC bé nhất khi bốn điểm B,M,M’,C’ thẳng hàng.

Khi đó góc BMA=1200, góc AMC=1200

Ta được vị trí của M trong tam giác ABC

  1. [Trường THPT Chuyên Biên Hòa- Tỉnh Hà Nam]

Cho đường tròn(O) có đường kính AB, P là điểm bất kì trên đường tròn, K là hình chiếu của P trên AB, R đối xứng với P qua AB. H bất kì trên AB. RH cắt lại (O) tại Q. Gọi đường tròn tâm I bán kính r tiếp xúc với HP,HQ và đường tròn (O)

Chứng minh: =+

Lời giải

Gọi T là tiếp điểm của (I) và HQ

J là điểm∈ IH sao cho HJ = HT

PQ cắt AB tại X, AP cắt BQ tại W

AQ cắt PB tại J’

⇒ J’ là trực tâm Δ WAB

H,J,I,W thẳng hàng

tantan(==

=

==-tan

=1800

Hai tam giác HKR và HTI đồng dạng với nhau nên ta có

LOẠI 8: Các bài toán khác

  1. [ ĐỀ THI CHỌN HỌC SINH GIỎI TOÁN TỈNH NAM ĐỊNH -2005]

Biết rằng số đo ba góc trong của tam giác lập thành một cấp số nhân với công bội . Gọi là đường tròn ngoại tiếp và là trọng tâm của tam giác .

1) Tính độ dài đoạn theo .

2) Biêt , hãy tính gần đúng số đo diện tích tam giác (lấy đến chữ số sau dấu phảy).

  1. [ ĐỀ THI CHỌN HỌC SINH GIỎI TOÁN TỈNH NAM ĐỊNH -2004]

Trên mặt phẳng cho tứ giác lồi có:

1) Nếu biết Hãy tính diện tích tứ giác theo .

2) Giả sử tứ giác thay đổi, mà không đổi. Hãy tìm giá trị lớn nhất của diện tích tứ giác

  1. [THI HỌC SINH GIỎI KHU VỰC DUYÊN HẢI VÀ ĐÒNG BẰNG BẮC BỘ NĂM HỌC 2013-2014]

Cho và một điểm nằm ngoài . Chứng minh rằng các đường chéo của bất kì hình thang nào nội tiếp mà hai cạnh bên kéo dài gặp nhau tại đều cắt nhau tại một điểm cố định

Hướng dẫn giải

Gọi là giao điểm của và

Ta có

Do

hay cố định.

  1. [TRẠI HÈ HÙNG VƯƠNG LẦN THỨ X TRƯỜNG THPT CHUYÊN LÊ QUÝ ĐÔN ĐIỆN BIÊN ]

Cho tam giác đều nội tiếp đường tròn . là một điểm di động trên đoạn ( khác ). Đường thẳng đi qua và vuông góc với cắt cung nhỏ tại . Gọi là hình chiếu của trên .

a) Các tiếp tuyến của tại và cắt tiếp tuyến tại của lần lượt tại . cắt lần lượt tại và . Chứng minh .

b) Tìm giá trị lớn nhất của chu vi tam giác theo

Lời giải

Có tứ giác nội tiếp

sđ;sđ

tứ giác nội tiếp

Từ và ta có năm điểmcùng nằm trên một đường tròn

và đồng dạng

hay

Trên đoạn lấy điểm sao cho

đều

Chu vi tam giác là

Đẳng thức xảy ra khi là đường kính của => là điểm chính giữa cung => là trung điểm đoạn

Vậy giá trị lớn nhất của chu vi tam giác là

Gọi là giao điểm của và

Giá trị lớn nhất của chu vi tam giác là .

  1. [TRẠI HÈ HÙNG VƯƠNG LẦN THỨ XI TRƯỜNG THPT CHUYÊN TỈNH SƠN LA ]

Cho tam giác vuông tại có là đường cao. là điểm tùy ý thuộc đoạn ( khác và ); gọi P là điểm thuộc kéo dài sao cho và là điểm thuộc tia kéo dài sao cho ; cắt tại . Chứng minh rằng

Hướng dẫn giải.

Gọi và lần lượt là giao điểm của , với đường tròn ngoại tiếp tam giác ; I là giao điểm của và .

là đường kính của nên, là hai đường cao của tam giác , do đó là trực tâm của tam giác thẳng hàng.

Suy ra hai tam giác , đồng dạng

Suy ra góc suy ra tứ giác là tứ giác nội tiếp.

Suy ra góc và (1)

Tương tự ta có: góc và . (2)

Từ (1) và (2) và ( do tứ giác nội tiếp) nên.

  1. [THI HỌC SINH GIỎI KHU VỰC DUYÊN HẢI VÀ ĐỒNG BẰNG BẮC BỘ NĂM HỌC 2013 - 2014 ]

Gọi là ba đường phân giác trong của tam giác vuông ở . Đoạn thẳng cắt tại . Đường thẳng qua song song với cắt lần lượt ở . Chứng minh rằng:

  1. [THI HỌC SINH GIỎI TỈNH THÁI NGUYÊN GIẢI TOÁN TRÊN MÁY TÍNH CASIO LỚP 11NĂM HỌC 2011-2012 ]

Qua một điểm nằm trong tam giác kẻ 3 đường thẳng song song với các cạnh của tam giác. Các đường thẳng này chia tam giác thành 6 phần, trong đó có 3 tam giác với các diện tích là ,; Tính diện tích của tam giác đã cho theo S1, S2, S3.

Hướng dẫn giải.

hay

Tương tự,

Từ đó

Suy ra

Hay

Thay số ta có:

  1. [THI HSG TRƯỜNG THPT TRƯNG VƯƠNG- BÌNH ĐỊNH 2006-2007]

T ìm điểm trong tam giác nhọn cho trước để bé nhất

Lời giải.

Dựng tam giác ngoại tiếp tam giáccó các cạnh tỷ lệ

nhìn các đoạn các góc bù góc

A

B

C

M

X

Y

Z

Sử dụng giao các cung chứa góc tìm được trong tam giác

  1. [ĐỀ THI HSG VÒNG 1TRƯỜNG THPT TRƯNG VƯƠNG- BÌNH ĐỊNH]

Cho các điểm nằm trên các cạnh của tam giác. Chứng minh rằng trong ba tam giác ,,có ít nhất một tam giác có diện tích nhỏ hơn hoặc bằng một phần tư diện tích tam giác .

Lời giải

.Tương tự có

Từ đó suy ra điều phải chứng minh

  1. [HỘI CÁC TRƯỜNG THPT CHUYÊN VÙNG DUYÊN HẢI & ĐỒNG BẰNG BẮC BỘ.TRƯỜNG THPT CHUYÊN LÊ KHIẾT – QUẢNG NGÃI NĂM 2016 ]

Cho tam giác thay đổi nhưng luôn là tam giác nhọn có tổng các bình phương các độ dài các cạnh là không đổi. Gọi là đường phân giác trong, lần lượt là hình chiếu vuông góc của trên , là giao điểm của , là giao điểm của và đường cao kẻ từ của tam giác . Tìm giá trị lớn nhất của tổng khi tam giác thay đổi.

Lời giải

Gọi là hình chiếu vuông góc của trên. Ta chứng minh thẳng hàng bằng cách chứng minh đồng quy.

Vì lần lượt thuộc các đoạn nên

A

E

K

F

C

A

D

B

Do đó theo định lý Ceva, đồng quy hoặc song song. Mà ba đường thẳng này không thể song song nên chúng đồng quy hay nằm trên đường cao của tam giác , do đó, là trực tâm của tam giác .

Gọi là hai đường cao còn lại của tam giác và lần lượt là độ

dài các cạnh thì theo tính chất tứ giác nội tiếp ta có

Tương tự, có

Lại có

nên

= a2 + b2 + c2.

Dấu “=” chỉ xảy ra khi , tức là tam giác đều

Vậy giá trị lớn nhất của tổng là .

  1. [Ngân hàng đề Hùng Vương-Trường CHUYÊN BẮC GIANG – năm-Tỉnh BẮC GIANG]

Cho tam giác nhọn không cân tại, là trung điểm cạnh, và tương ứng là chân đường cao hạ từ, của tam giác. là giao điểm của hai tiếp tuyến tại , của đường tròn ngoại tiếp tam giác, là giao điểm của và, là giao điểm của và

a) Chứng minh rằng là tâm đường tròn nội tiếp tam giác .

b) Đường tròn ngoại tiếp tam giác và cắt nhau tại điểm thứ hai . Chứng minh rằng trực tâm của tam giác nằm trên.

c) Chứng minh rằng D nằm trên đường tròn ngoại tiếp tam giác XYZ.

Hướng dẫn giải

a) là phân giác góc.

Do và , từ đó và nằm trên trung trực của, do đó là tâm đường tròn nội tiếp tam giác

b)Giả sử, khi đó nằm trên cung nhỏ. Gọi là tâm đường tròn ngoại tiếp tam giác và là trung điểm của. Ta có được và vuông góc.

Từ và vuông góc, ta được và song song. là hình bình hành nên song song với, do đó , , thẳng hàng.

c) Chứng minh được góc và là đường trung trực của .

Chứng minh được góc nên là tứ giác nội tiếp.

Do đó góc , do đó là tứ giác nội tiếp.

  1. [KỲ THI OLIMPIC HÙNG VƯƠNG LẦN THỨ X- NĂM 2014- TRƯỜNG CHUYÊN HÀ GIANG]

Các đường phân giác trong của tam giác có chu vi cắt các đoạn thẳng tương ứng tại A. Các đường thẳng qua A2 song song với BC cắt AB, AC theo thứ tự tại A3, A4. Đường thẳng qua B2 song song với AC cắt BC, BA theo thứ tự tại B3, B4. Đường thẳng qua C2 song song với AB cắt CA, CB theo thứ tự tại C3, C4. Chứng minh rằng:

AB4+BC4+CA4+BA3+CB3+AC3 p

Đẳng thức xảy ra khi nào?

Hướng dẫn giải

Tải tài liệu này file docx word pdf